All Exams  >   UPPSC (UP)  >   Course for UPPSC Preparation  >   All Questions

All questions of Reading Comprehension for UPPSC for UPPSC (UP) Exam

Those who opine lose their impunity when the circumstances in which they pontificate are such that generate from their expression a positive instigation of some mischievous act. An opinion that corn dealers are starvers of the poor, or that owning private property is robbery, ought to be unmolested when simply circulated through the press, but may justly incur punishment when delivered orally to an excited mob assembled before the house of a corn dealer, or when handed about among the same mob in the form of a placard. Acts, of whatever kind, which without justifiable cause do harm to others, may be, and in the more important cases are absolutely required to be, controlled by the unfavourable sentiments, and, when needful, by the active interference of mankind. The liberty of the individual must be thus far limited; he must not make himself a nuisance to other people. But if he refrains from molesting others in matters that concern them, and merely acts according to his own inclination and judgment in matters which concern himself he should be allowed, without molestation, to carry his opinions into practice at his own cost. As it is useful that while mankind are imperfect there should be different opinions, so it is that there should be different experiments of living, that free scope should be given to varieties of character, short of injury to others, and that the worth of different modes of life should be proved practically, when anyone thinks fit to try them. Where not the person‘s own character but the traditions and customs of other people are the rule of conduct, there is wanting one of the principal ingredients of individual and social progress.  
It would be absurd to pretend that people ought to live as if nothing whatever had been known in the world before they came into it; as if experience had as yet done nothing toward showing that one mode of existence, or of conduct, is preferable to another. Nobody denies that people should be so taught and trained in youth as to know and benefit by the ascertained results of human experience. But it is the privilege and proper condition of a human being, arrived at the maturity of his faculties, to use and interpret experience in his own way. It is for him to find out what part of recorded experience is properly applicable to his own circumstances and character. The traditions and customs of other people are, to a certain extent, evidence of what their experience has taught them—presumptive evidence, and as such, have a claim to his deference—but, in the first place, their experience may be too narrow, or they may have not interpreted it rightly. Secondly, their interpretation of experience may be correct, but unsuited to him. Customs are made for customary circumstances and customary characters, and his circumstances or his character may be uncustomary. Thirdly, though the customs be both good as customs and suitable to him, yet to conform to custom merely as custom does not educate him or develop in him any of the qualities which are the distinctive endowments of a human being. He gains no practice either in discerning or desiring what is best. 
Directions: Read the above paragraph and answer the following
Q.The author holds that one should not necessarily defer to the traditions and customs of other people. The author supports his position by arguing that:  
I. traditions and customs are usually the result of misinterpreted experiences.  
II. customs are based on experiences in the past, which are different from modern experiences.
III. customs can stifle one‘s individual development. 
  • a)
    II only  
  • b)
    III only  
  • c)
    I and III only  
  • d)
    II and III only  
  • e)
    None of the above 
Correct answer is option 'B'. Can you explain this answer?

Moumita Rane answered
Mapping the Passage:
¶1 (first half) introduces the limits that should be placed on expression of opinions.
¶1 (second half) argues that differences of opinions are necessary.
¶2 offers three reasons why individual opinions are necessary to personal growth.
Use your reasoning from the last question to help yourself with this one. Remember to eliminate answer choices as you either select or eliminate Roman numerals. RN III is in three choices, so look there first. The author argues that customs aren‘t always good, and therefore can stifle growth. RN III is correct. Eliminate (A). Next to RN II: it also contradicts the author‘s argument that customs can be useful. By default, the answer must be (B), but check RN I to be sure. The author defends the general usefulness of customs. While sometimes experiences are misinterpreted, there‘s nothing to indicate that the usually are, which knocks RN I out immediately. RN II.

The golden toad of Costa Rica, whose beauty and rarity inspired an unusual degree of human interest from a public generally unconcerned about amphibians, may have been driven to extinction by human activity nevertheless. In the United States, a public relations campaign featuring the toad raised money to purchase and protect the toad’s habitat in Costa Rica, establishing the Monteverde Cloud Forest Preserve in 1972. Although this action seemed to secure the toad’s future, it is now apparent that setting aside habitat was not enough to save this beautiful creature. The toad’s demise in the late 1980s was a harbinger of further species extinction in Costa Rica. Since that time, another twenty of the fifty species of frogs and toads known to once inhabit a 30 square kilometer area near Monteverde have disappeared.
The unexplained, relatively sudden disappearance of amphibians in Costa Rica is not a unique story. Populations of frogs, toads, and salamanders have declined or disappeared the world over. Scientists hypothesize that the more subtle effects of human activities on the world’s ecosystems, such as the build-up of pollutants, the decrease in atmospheric ozone, and changing weather patterns due to global warming, are beginning to take their toll. Perhaps amphibians - whose permeable skin makes them sensitive to environmental changes - are the “canary in the coal mine,” giving us early notification of the deterioration of our environment. If amphibians are the biological harbingers of environmental problems, humans would be wise to heed their warning.
Q. 
The passage implies that
  • a)
    many amphibians are not considered beautiful.
  • b)
    the Monteverde Cloud Forest Preserve was not large enough to protect the golden toad.
  • c)
    only Costa Rican amphibians living near Monteverde have disappeared since the 1980s.
  • d)
    amphibians sometimes live in coal mines.
  • e)
    no humans yet consider the decline of amphibious populations an indication of a threat to human populations.
Correct answer is option 'A'. Can you explain this answer?

Explanation:

Amphibians are not considered beautiful:
- The passage mentions that the golden toad of Costa Rica was an exception to the general lack of human interest in amphibians.
- The fact that a public relations campaign was needed to raise money for the toad's habitat suggests that beauty alone was not enough to garner public support.
- The rarity of the golden toad also played a significant role in generating human interest, indicating that beauty alone may not have been the primary factor.
Therefore, the passage implies that many amphibians are not considered beautiful, as evidenced by the lack of public interest in amphibians in general.

For years, U.S. employers have counted on a steady flow of labor from Mexico willing to accept low-skilled, low paying jobs. These workers, many of whom leave economically depressed villages in the Mexican interior, are often more than willing to work for wages well below both the U.S. minimum wage and the poverty line.
However, thanks to a dramatic demographic shift currently taking place in Mexico, the seemingly inexhaustible supply of workers migrating from Mexico to the United States might one day greatly diminish if not cease.
Predictions of such a drastic decrease in the number of Mexican immigrants, both legal and illegal, are driven by Mexico’s rapidly diminishing population growth. As a result of a decades-long family planning campaign, most Mexicans are having far fewer children than was the norm a generation ago. The campaign, organized around the slogan that “the small family lives better,” saw the Mexican government establish family-planning clinics and offer free contraception. For nearly three decades, the government’s message concerning population hasn’t wavered. In fact, the Mexican Senate recently voted to extend public school sex education programs to kindergarten.
The result of Mexico’s efforts to stem population growth is nothing short of stunning. In 1968, the average Mexican woman had just fewer than seven children; today, the figure is slightly more than two. For two primary reasons, Mexico’s new demographics could greatly impact the number of Mexicans seeking work in the U.S. First, smaller families by their nature limit the pool of potential migrants.
Second, the slowing of Mexico’s population growth has fostered hope that Mexico will develop a healthy middle class of people content to make their livelihoods in their home country.
Though the former of these factors is all but assured, the growth of a healthy middle class is far from a foregone conclusion. The critical challenge for Mexico is what it does with the next 20 years. Mexico must invest in education, job training, and infrastructure, as well as a social-security system to protect its aging population.  If Mexico is willing to step forward and meet this challenge, America may one day wake up to find that, like cheap gasoline, cheap Mexican labor has become a thing of the past.
Q.
One function of the final paragraph of the passage is to
  • a)
    relate why the number of Mexican immigrants seeking work in the United States is certain to decline.
  • b)
    detail the successes of Mexico’s family planning campaign
  • c)
    explain why the number of Mexican immigrants seeking work in the United States may not dramatically decrease.
  • d)
    specify the types of infrastructure in which Mexico must invest.
  • e)
    notify American employers that they will soon need to find alternative sources of labor.
Correct answer is option 'C'. Can you explain this answer?

Arya Yadav answered
Explanation:
Why the number of Mexican immigrants seeking work in the United States may not dramatically decrease:
- The final paragraph of the passage discusses the challenges that Mexico faces in developing a healthy middle class.
- It highlights the need for Mexico to invest in education, job training, infrastructure, and social security to support its population.
- This suggests that while the decrease in Mexican immigrants seeking work in the U.S. is possible, it is not guaranteed.
- If Mexico successfully addresses these challenges, it may not lead to a dramatic decrease in the number of Mexican immigrants seeking work in the United States.
Conclusion:
- Therefore, the function of the final paragraph is to explain why the number of Mexican immigrants seeking work in the United States may not dramatically decrease, emphasizing the importance of Mexico's actions in the coming years.

While many points are worth making in an evaluation of the single sixyear presidential term, one of the most telling points against the single term has not been advanced. This kind of constitutional limitation on elections is generally a product of systems with weak or non-existent political parties.
Since there is no party continuity or corporate party integrity in such systems, there is no basis for putting trust in the desire for re-election as a safeguard against mismanagement in the executive branch. Better under those conditions to operate on the basis of negative assumptions against incumbents. I do not know if the earliest proposal for a single, nonrepeatable term was made in the 1820s because that was a period of severely weak political parties. But I do feel confident that this is a major reason, if not the only reason, that such a proposal has been popular since the 1940s.
Though the association of the non-repeatable election with weak political parties is not in itself an argument against the limitation, the fallout from this association does contribute significantly to the negative argument. Single-term limitations are strongly associated with corruption. In any weak party system, including the presidential system, the onus of making deals and compromises, both shady and honourable, rests heavily upon individual candidates. Without some semblance of corporate integrity in a party, individual candidates have few opportunities to amortize their obligations across the spectrum of elective and appointive jobs and policy proposals. The deals tend to be personalized and the payoffs come home to roost accordingly.
If that situation is already endemic in conditions of weak or nonexistent parties, adding to it the limitation against re-election means that candidates and officials, already prevented from amortizing their deals across space, are also unable to amortize their obligations temporally. This makes for a highly beleaguered situation. The single six-year term for presidents is an effort to compensate for the absence of a viable party system, but it is a compensation ultimately paid for by further weakening the party system itself.
 Observers, especially foreign observers, have often noted that one source of weakness in American political parties is the certainty of election every two or four years, not only because any artificial limitation on elections is a violation of democratic principles but also because when elections are set in a certain and unchangeable cycle, political parties do not have to remain alert but can disappear into inactivity until a known point prior to the next election. To rigidify matters by going beyond the determinacy of the electoral cycle to add an absolute rule of one term would hang still another millstone around the neck of already doddering political parties.  
Directions: Read the above paragraph and answer the following:
Q. Which of the following, if true, would most weaken the author‘s claim about single-term political systems?  
  • a)
    The discovery that foreign observers like this system          
  • b)
    The discovery that most politicians are honest          
  • c)
    The discovery that Americans dislike this system            
  • d)
    The discovery that parliamentary systems are more democratic              
  • e)
     The discovery that politicians favour such a system 
Correct answer is option 'B'. Can you explain this answer?

Saanvi Sen answered
Explanation:

Analysis of the question:
The author argues that one of the main drawbacks of single-term political systems is the lack of party continuity and corporate party integrity, which can lead to corruption and inefficiency. To weaken this claim, we need to find a scenario that contradicts the author's assertion.

Key point:
- The author argues that weak political parties in single-term systems contribute to corruption and inefficiency.

Explanation of the correct answer:
- If it is discovered that most politicians in single-term systems are honest, it would weaken the author's claim about corruption and inefficiency being inherent in such systems. This would suggest that the lack of party continuity and corporate party integrity does not necessarily lead to negative consequences.

Explanation of incorrect answers:
a) The opinion of foreign observers about the system does not directly impact the author's argument about corruption and inefficiency.
c) The dislike of Americans for the system does not address the author's argument about corruption and inefficiency.
d) The comparison to parliamentary systems being more democratic does not address the author's argument about corruption and inefficiency.
e) The preference of politicians for such a system does not address the author's argument about corruption and inefficiency.

Direction For question : As the Heart of Darkness makes its way into the savage shadows of the African continent, Joseph Conrad exposes a psycho-geography of the collective unconscious in the entangling symbolic realities of the serpentine Congo. Conrad’s novella descends into the unknowable darkness at the heart of Africa, taking its narrator, Marlow, on an underworld journey of individuation, a modern odyssey toward the center of the Self and the center of the Earth. Ego dissolves into soul as, in the interior; Marlow encounters his double in the powerful image of ivory-obsessed Kurtz, the dark shadow of European imperialism. The dark meditation is graced by personifications of anima in Kurtz’ black goddess, the savagely magnificent consort of the underworld, and in his porcelain -skinned Persephone, innocent intended of the upper world. “Africa,” wrote Graham Greene, “will always be the Africa in the Victorian atlas, the blank unexplored continent in the shape of the human heart.” The African heart described by Greene “acquired a new layer of meaning when Conrad portrayed the Congo under King Leopold as the Heart of Darkness, a place where barbarism triumphs over humanity, nature over technology, biology over culture, id over super ego.” The unknown and uncharted topography of the African continent first beckoned Conrad’s narrator, Marlow, into its depths in his boyhood: “Now, when I was a little chap I had a passion for maps. I would look for hours at South America, or Africa, or Australia, and lose myself in all the glories of exploration”. When Marlow was grown and Africa was no longer a blank space on the map, but rather “a place of darkness,” there was still one river there that drew him especially, “a mighty big river, that you could see on the map, resembling an immense snake uncoiled, with its head in the sea, its body at rest curving afar over a vast country, and its tail lost in the depths of the land”. This same deep place, the Congo, that had seduced Conrad’s ivory hunting Kurtz into the horrors of its savage embrace had, in 1890, lured Conrad himself into adventure that turned him from sailor to writer and severely affected his health for the rest of his life. As the voyage up the Congo pro ved fateful for the development of Conrad’s narrator, Marlow, it was equally fateful for Conrad’s individuation, as he reflects in his letters “Before the Congo I was just a mere animal.” Hillman, in “Notes on White Supremacy” reminds us that, like Conrad, both Freud and Jung were called to venture into the shadowed continent and vestiges of their journeys still colour our psychological language: The convention informing geographical discoveries and the expansion of white consciousness over Africa continue to inform psychic geography. The topological language used by Freud for “the unconscious” as a place below, different, timeless, primordial, libidinal and separated from the consciousness recapitulates what white reporters centuries earlier said about West Africa. From Conrad’s Heart of Darkness to van der Post’s Venture to the Interior, Africa and the unconscious allegorize the other place.... “Just don’t stay in the topical colonies too long; you must reign at home,” writes Freud in 1911 to Jung, who himself made the African journey fourteen years later, describing the vast lands and dark peoples he encountered in language he applies as well to the immemorial unconscious psyche.... Part of psychology’s myth is that the unconscious was “discovered” as its contents are “explored”. Thus Africa has become a topology of the mind — its location, its shape, its cultures, its textures, its rhythms, its foliage, its hues, its wildness — all calling forth something lost in the psychology of the white European. It is with an understanding of our destiny to explore that symbolic lost continent within ourselves that we can begin to appreciate the prescience of Jungian psychology in Conrad’s Heart of Darkness.
Q. The author compares Congo to an animal/a reptile. What is this figure of speech called?
  • a)
    Onomatopoeia
  • b)
    Metaphor
  • c)
    Alliteration
  • d)
    Euphemism
Correct answer is option 'B'. Can you explain this answer?

Aditya Kumar answered
Metaphor is a figure of speech that means implied comparison. The author uses the metaphor of a snake to describe Congo. Alliteration is the repetition of the beginning sounds of neighboring words. Euphemism is a mild, indirect, or vague term substituting for a harsh, blunt, or offensive term. Onomatopoeia is the formation of a word from a sound associated with what is named.

Measuring more than five feet tall and ten feet long, the Javan rhinoceros is often called the rarest large mammal on earth. None exist in zoos. Like the Indian rhino, the Javan has only one horn; African and Sumatran rhinos have two. While the Javan rhino habitat once extended across southern Asia, now there are fewer than one hundred of the animals in Indonesia and under a dozen in Vietnam. Very little is known about Javan rhinos because they lead secretive and solitary lives in remote jungles.
Until recently, scientists debated whether females even have horns, and most scientific work has had to rely on DNA garnered from dung.
The near extinction of the Javan rhino is the direct result of human actions. For centuries, farmers, who favored the same habitat, viewed them as crop eating pests and shot them on sight. During the colonial period, hunters slaughtered thousands. Now, human efforts to save them may well prove futile. The Vietnamese herd is probably doomed, as too few remain to maintain the necessary genetic variation. Rhinos from Java cannot supplement the Vietnamese numbers because in the millions of years since Indonesia separated from the mainland, the two groups have evolved into separate sub-species. In Indonesia, the rhinos are protected on the Ujung Kulon peninsula, which is unsettled by humans, and still have sufficient genetic diversity to have a chance at survival.
Ironically, however, the lack of human disturbance allows mature forests to replace the shrubby vegetation the animals prefer. Thus, human benevolence may prove little better for these rhinos than past human maltreatment.
Q.
Which of the following can be inferred from the passage?
  • a)
    Javan rhinos are one of the most endangered animals on the planet.
  • b)
    More is known about the genetics of the Javan rhino than is known about its mating patterns.
  • c)
    Hunters killed more Javan rhinos in Vietnam than in Indonesia.
  • d)
    Most animal extinctions are the result of human actions.
  • e)
    Genetic diversity is the most important factor for the survival of a species.
Correct answer is option 'B'. Can you explain this answer?

The correct answer to this question will be a claim that must follow from text contained in the passage. This question does not provide any clues as to where the justifying text will be found.
(A) The first paragraph mentions that the the Javan rhino is the most endangered species of large mammal. However, it does not have to be true that it is one of the most endangered animals, a category that is far broader than mammals.
(B) CORRECT. The first paragraph states that very little is known about the life of the Javan rhino. The passage does indicate, however, that scientists have been able to extract information on the species’ DNA from gathered dung. The paragraph also suggests that very little information about female Javan rhinos has been gained, given that scientists only recently discovered whether or not females of the species even have horns. Thus, one can infer that more is known about the genetics of the Javan rhino than its mating patterns.
(C) The second paragraph indicates that hunters slaughtered many rhinos, but it does not mention where. Furthermore, it does not have to be true that more rhinos were killed in Vietnam simply because fewer rhinos remain there.
(D) The passage only discusses the Javan rhino; therefore, a generalization about the extinction of "most animals" is not supported by the passage.
(E) The passage does not mention other survival factors for a species or rank them; therefore, this inference is not supported by the passage. 

In all battles two things are usually required of the Commander-in-Chief: to make a good plan for his army and to keep a strong reserve. Both of these are also obligatory for the painter. To make a plan, thorough reconnaissance of the country where the battle is to be fought is needed. Its fields, its mountains, its rivers, its bridges, its trees, its flowers, its atmosphere—all require and repay attentive observation from a special point of view.
I think this is one of the chief delights that have come to me through painting. No doubt many people who are lovers of art have acquired it to a high degree without actually practicing. But I expect that nothing will make one observe more quickly or more thoroughly than having to face the difficulty of representing the thing observed. And mind you, if you do observe accurately and with refinement, and if you do record what you have seen with tolerable correspondence, the result follows on the canvas with startling obedience.
But in order to make his plan, the General must not only reconnoitre the battle-ground; he must also study the achievements of the great Captains of the past. He must bring the observations he has collected in the field into comparison with the treatment of similar incidents by famous chiefs.
Considering this fact, the galleries of Europe take on a new—and to me at least — a severely practical interest. You see the difficulty that baffled you yesterday; and you see how easily it has been overcome by a great or even by a skilful painter. Not only is your observation of Nature sensibly improved and developed, but also your comprehension of the masterpieces of art.
But it is in the use and withholding of their reserves that the great commanders have generally excelled. After all, when once the last reserve has been thrown in, the commander‘s part is played. If that does not win the battle, he has nothing else to give. Everything must be left to luck and to the fighting troops. But these last reserves, in the absence of high direction, are apt to get into sad confusion, all mixed together in a nasty mess, without order or plan—and consequently without effect.  
Mere masses count no more. The largest brush, the brightest colours cannot even make an impression. The pictorial battlefield becomes a sea of mud mercifully veiled by the fog of war. Even though the General plunges in himself and emerges bespattered, as he sometimes does, he will not retrieve the day. In painting, the reserves consist in Proportion or Relation. And it is here that the art of the painter marches along the road which is traversed by all the greatest harmonies in thought. At one side of the palette there is white, at the other black; and neither is ever used 'neat.‘ Between these two rigid limits all the action must lie, all the power required must be generated. Black and white themselves placed in juxtaposition make no great impression; and yet they are the most that you can do in pure contrast.  
Directions: Read the above paragraph and answer the following
Q.Following the example of the master Manet, the young Matisse often inserted in his pictures areas of white such as tablecloths or crockery that allowed for striking contrasts with black objects such as a knife or a dark bottle. What is the relevance of this information to the passage?  
  • a)
    It supports the author‘s claim that the great artists are worthy of imitation.  
  • b)
    It supports the author‘s claim that neither black nor white is ever used ‗neat.‘
  • c)
    It weakens the author‘s claim that black and white themselves placed in juxtaposition make no great impression.  
  • d)
    It weakens the author‘s claim that great painters take Nature as their subject.  
  • e)
    This information has no relevance to the information in the passage 
Correct answer is option 'C'. Can you explain this answer?

Rithika Kaur answered
Mapping the Passage
¶1 compares painting a picture to fighting a battle, lists two similarities, planning and
backup, and discusses planning.
¶2 explains that practicing art is a great way to become a lover of art.
¶s3 and 4 draws in the analogy of the general and explain the need to study previous masters in war and art.
¶s5 and 6 explain the need to keep reserves in battle and painting.
A synthesis question testing your ability to evaluate the relevance of a new situation to the author‘s arguments. Zero in on elements of the new situation that sound relevant to the passage. Black and white are mentioned in the final paragraph. Recall that the author argues that black and white make weak impressions when contrasted. However, in the question stem situation, the impression is strong. We‘re looking for an answer that points this out, in other words, one that argues the new situation weakens the author‘s view (C) fits exactly.

In all battles two things are usually required of the Commander-in-Chief: to make a good plan for his army and to keep a strong reserve. Both of these are also obligatory for the painter. To make a plan, thorough reconnaissance of the country where the battle is to be fought is needed. Its fields, its mountains, its rivers, its bridges, its trees, its flowers, its atmosphere—all require and repay attentive observation from a special point of view.
I think this is one of the chief delights that have come to me through painting. No doubt many people who are lovers of art have acquired it to a high degree without actually practicing. But I expect that nothing will make one observe more quickly or more thoroughly than having to face the difficulty of representing the thing observed. And mind you, if you do observe accurately and with refinement, and if you do record what you have seen with tolerable correspondence, the result follows on the canvas with startling obedience.
But in order to make his plan, the General must not only reconnoitre the battle-ground; he must also study the achievements of the great Captains of the past. He must bring the observations he has collected in the field into comparison with the treatment of similar incidents by famous chiefs.
Considering this fact, the galleries of Europe take on a new—and to me at least — a severely practical interest. You see the difficulty that baffled you yesterday; and you see how easily it has been overcome by a great or even by a skilful painter. Not only is your observation of Nature sensibly improved and developed, but also your comprehension of the masterpieces of art.
But it is in the use and withholding of their reserves that the great commanders have generally excelled. After all, when once the last reserve has been thrown in, the commander‘s part is played. If that does not win the battle, he has nothing else to give. Everything must be left to luck and to the fighting troops. But these last reserves, in the absence of high direction, are apt to get into sad confusion, all mixed together in a nasty mess, without order or plan—and consequently without effect.  
Mere masses count no more. The largest brush, the brightest colours cannot even make an impression. The pictorial battlefield becomes a sea of mud mercifully veiled by the fog of war. Even though the General plunges in himself and emerges bespattered, as he sometimes does, he will not retrieve the day. In painting, the reserves consist in Proportion or Relation. And it is here that the art of the painter marches along the road which is traversed by all the greatest harmonies in thought. At one side of the palette there is white, at the other black; and neither is ever used 'neat.‘ Between these two rigid limits all the action must lie, all the power required must be generated. Black and white themselves placed in juxtaposition make no great impression; and yet they are the most that you can do in pure contrast.  
Directions: Read the above paragraph and answer the following
Q.As the author creates the analogy between war and painting in the passage, the Commander-in-Chief is to the battleground as the:  
  • a)
    painter is to the subject being painted.  
  • b)
    painter is to the canvas of the painting.  
  • c)
    painter is to the paint colours.
  • d)
    painter is to the art gallery.  
  • e)
    painter is to the brush 
Correct answer is option 'A'. Can you explain this answer?

Mapping the Passage
¶1 compares painting a picture to fighting a battle, lists two similarities, planning and
backup, and discusses planning.
¶2 explains that practicing art is a great way to become a lover of art.
¶s3 and 4 draws in the analogy of the general and explain the need to study previous masters in war and art.
¶s5 and 6 explain the need to keep reserves in battle and painting.
The Commander-in-Chief is mentioned in the first paragraph, so begin your search there. The author says that the battleground must be inspected and studied. What is the equivalent in painting? The subject being painted. (A) fits the bill.

As formal organizations, business corporations are distinguished by their particular goals, which include maximization of profits, growth, and survival. Providing goods and services is a means to this end. If, for example, a number of individuals (outsiders or even insiders) believe that a company‘s aggressive marketing of infant formula in third world countries is morally wrong, the company is unlikely to be moved by arguments based on ethos alone as long as what it is doing remains profitable. But if those opposed to the company‘s practice organize a highly effective boycott of the company‘s products, their moral views will soon enter into the company‘s deliberations indirectly as limiting operating conditions. They can, at this point, no more be ignored than a prohibitive increase in the costs of certain raw materials.  
Although the concepts and categories of ethics may be applied to the conduct of corporations, there are important differences between the values and principles underlying corporate behaviour and those underlying the actions of most individuals. If corporations are by their nature end- or goal-directed how can they acknowledge acts as wrong in and of themselves? Is it possible to hold one criminally responsible for acts that if performed by a human person would result in criminal liability?  
The first case of this type to achieve widespread public attention was the attempt to prosecute the Ford Motor Company for manslaughter as the result of alleged negligent or reckless decision making concerning the safety engineering of the Pinto vehicle. Although the defendant corporation and its officers were found innocent after trial, the case can serve as an exemplar for our purposes.  
In essence, the prosecution in this case attempted to show that the corporation had produced and distributed a vehicle that was known to be defective at the time of production and sale, and that even after a great deal of additional information accumulated regarding the nature of the problems, the corporation took no action to correct them. The obvious non-corporate analogy would be the prosecution of a person who was driving a car with brakes known to be faulty, who does not have them repaired because it would cost too much, and who kills someone when the brakes eventually fail and the car does not stop in time. Such cases involving individuals are prosecuted and won regularly.
If corporations have no concept of right or wrong because they are exclusively goal-directed, can they be convicted in cases of this type, and what purpose would be served by such a conviction? Perhaps we can make a utilitarian argument for convicting corporations of such crimes. The argument would be that of deterrence; conviction and punishment would deter other corporations from taking similar actions under similar circumstances. However, there appears to be considerable evidence that deterrence does not work on corporations, even if, arguably, it works on individuals. The possibility of being discovered and the potential magnitude of the fine merely become more data to be included in the analysis of limiting conditions.  
Directions: Read the above paragraph and answer the following:
Q.A claim that things have ethical value to corporations only insofar as they are instrumental in furthering the ultimate goals of the corporation is:  
  • a)
    necessarily true, given the information presented in the passage.  
  • b)
    perhaps true, and supported by the information presented in the passage.  
  • c)
    perhaps true, but not supported by any information in the passage.  
  • d)
    necessarily false, given the information presented in the passage.  
  • e)
    a figment of the author‘s imagination
Correct answer is option 'B'. Can you explain this answer?

Arpita Basu answered
Explanation:

Claim about ethical value to corporations:
- The claim that things have ethical value to corporations only insofar as they are instrumental in furthering the ultimate goals of the corporation is perhaps true, and it is supported by the information presented in the passage.
- The passage discusses how corporations are distinguished by their goals of profit maximization, growth, and survival, and how providing goods and services is a means to achieve these goals.
- It also mentions that corporations may not be moved by ethical arguments alone if their actions remain profitable, indicating that ethical considerations are secondary to their goals.

Support from the passage:
- The passage states that corporations are goal-directed entities, focused on achieving their objectives, which may not inherently include ethical values.
- It discusses how corporations may only consider moral views when they become limiting factors, similar to cost increases in raw materials.
- The example of the Ford Motor Company case further illustrates how corporations prioritize goals over ethical considerations.
Therefore, based on the information presented in the passage, the claim that ethical value to corporations is instrumental in furthering their goals is perhaps true and supported by the context provided.

Direction for question: Typewriters are the epitome of a technology that has been comprehensively rendered obsolete by the digital age. The ink comes off the ribbon, they weigh a ton, and second thoughts are a disaster. But they are also personal, portable and, above all, private. Type a document and lock it away and more or less the only way anyone else can get it is if you give it to them. That is why the Russians have decided to go back to typewriters in some government offices, and why in the US, some departments have never abandoned them. Yet it is not just their resistance to algorithms and secret surveillance that keeps typewriter production lines – well one, at least – in business (the last British one closed a year ago). Nor is it only the nostalgic appeal of the metal body and the stout well-defined keys that make them popular on eBay. A typewriter demands something particular: attentiveness. By the time the paper is loaded, the ribbon tightened, the carriage returned, the spacing and the margins set, there’s a big premium on hitting the right key. That means sorting out ideas, pulling together a kind of order and organising details before actually striking off. There can be no thinking on screen with a typewriter. Nor are there any easy distractions. No online shopping. No urgent emails. No Twitter. No need even for electricity – perfect for writing in a remote hideaway. The thinking process is accompanied by the encouraging clack of keys, and the ratchet of the carriage return. Ping!
Q. Which one of the following best describes what the passage is trying to do?
  • a)
    It describes why people continue to use typewriters even in the digital age.
  • b)
    It argues that typewriters will continue to be used even though they are an obsolete technology.
  • c)
    It highlights the personal benefits of using typewriters.
  • d)
    It shows that computers offer fewer options than typewriters.
Correct answer is option 'A'. Can you explain this answer?

Vikas Kapoor answered
Option 1 is the correct answer as the author provides examples of the US and Russia to tell that these countries have taken up the use of typewriter. The author also supports their using of typewriters by providing us with positive aspects of the typewriters. Option 2 is beyond the scope of the passage. Option 3 is incorrect since clearly it is not the main aim of the passage. There is no hint to make that claim. Option 4 is incorrect since this difference has nowhere been made.

The ―paradox of tolerance‖ admonishes us that tolerance of the intolerant leads to intolerance. The Universal Declaration of Human Rights and the constitutions and laws of Western European democracies that adhere to the principle of freedom of speech all heed the warning of this conundrum and do not afford legal protection to extremist speech.  
While in Western European democracies, the speech of nondemocratic extremists has been successfully outlawed, in the United States the first amendment right to freedom of speech has been interpreted to encompass radical oration. The traditional justifications of this American stance originate in the belief that speech is entitled to greater tolerance than other kinds of activity. They are based on the belief that speech itself is valuable, and thus ascribe positive value to a very broad range of speech.
 According to the classical model, freedom of speech serves an indispensable function in the process of democratic self-government. From this perspective, the free speech principle need only protect political speech, comprised of all the facts, theories, and opinions relating to any issue on which the citizens must vote. Proponents of this view insist that even extremist views cannot be concealed from voting citizens, if these views bear on any public issue before them.
Protection of free speech serves the collective self-interests of a selfgoverning society made up of all rational, equal, and fully participating citizens who take their civic duties seriously. The fortress model is built on a foundation of pessimism, individualism, relativism, and self-doubt. At its deepest level, the fortress model values freedom of speech as a necessary precondition to the discovery and preservation of truth, but even at this level the function of speech remains primarily negative.  
From this perspective, the government and a majority of the people pose a great danger of intolerance. In spite of the high probability that their beliefs will eventually prove to be false, it is argued, people nonetheless tend to feel certain about them and, consequently, feel justified in requiring others to conform. Thus, the fortress model‘s prescription for combating the tendency to censor nonconforming views is to overprotect speech by providing a broad ―buffer zone‖ that encompasses extremist speech because its protection substantially diminishes the probability that inherently valuable speech will be suppressed.  
Directions: Read the above paragraph and answer the following
Q. Which of the following scenarios, if true, would most weaken the argument contained in the paradox of tolerance which ―admonishes us that tolerance of the intolerant leads to intolerance?‖ 
  • a)
    Islandia‘s government has decided to outlaw extremist political groups in order to protect its democratic political system.  
  • b)
    Islandia has a non-democratic government, despite its suppression of extremist political groups.  
  • c)
    Islandia‘s government became democratic only after extremist political groups were outlawed.  
  • d)
    Islandia has had a stable democratic government for decades, even though it has never outlawed extremist political groups  
  • e)
    Islandia has tried and failed repeatedly to have a stable government 
Correct answer is option 'D'. Can you explain this answer?

Ameya Sen answered
Mapping the Passage
¶1 introduces the ―"paradox of intolerance."
¶2 gives examples of countries that have taken the paradox into consideration by banning extreme speech, and argues that the United States tolerates even extremist speech.
¶3 introduces a justification for the defense of extremist speech: the classical model.
¶s4 and 5 introduce another justification: the fortress model.
Take a moment to review the paradox and its implications before attacking the choices. The paradox essentially states that free speech should be limited at its extremes when the extremes could contribute to eliminating free speech. Looking for an answer choice that weakens the implications of the paradox turns up (D): The paradox states that extreme speech weakens stable government, a point weakened by a scenario in which extreme speech and stable government coexist.

Direction For Question : Enlightenment is man’s emergence from his self- imposed nonage. Nonage is the inability to use one’s own understanding without another’s guidance. This nonage is self-imposed if its cause lies not in lack of understanding but in indecision and lack of courage to use one’s own mind without another’s guidance. Dare to know! (Sapere aude.) “Have the courage to use your own understanding,” is therefore the motto of the enlightenment. Laziness and cowardice are the reasons why such a large part of mankind gladly remain minors all their lives, long after nature has freed them from external guidance. They are the reasons why it is so easy for others to set themselves up as guardians. It is so comfortable to be a minor. If I have a book that thinks for me, a pastor who acts as my conscience, a physician who prescribes my diet, and so on—then I have no need to exert myself. I have no need to think, if only I can pay; others will take care of that disagreeable business for me. Thus it is very difficult for the individual to work himself out of the nonage which has become almost second nature to him. He has even grown to like it, and is at first really incapable of using his own understanding because he has never been permitted to try it. Dogmas and formulas, these mechanical tools designed for reasonable use—or rather abuse— of his natural gifts, are the fetters of an everlasting nonage. The man who casts them off would make an uncertain leap over the narrowest ditch, because he is not used to such free movement. That is why there are only a few men who walk firmly, and who have emerged from nonage by cultivating their own minds. It is more nearly possible, however, for the public to enlighten itself; indeed, if it is only given freedom, enlightenment is almost inevitable. There will always be a few independent thinkers, even among the self- appointed guardians of the multitude. Once such men have thrown off the yoke of nonage, they will spread about them the spirit of a reasonable appreciation of man’s value and of his duty to think for himself. It is especially to be noted that the public which was earlier brought under the yoke by these men, afterwards forces these very guardians to remain in submission, if it is so incited by some of its guardians who are themselves incapable of any enlightenment. That shows how pernicious it is to implant prejudices: they will eventually revenge themselves upon their authors or their authors’ descendants. Therefore, a public can achieve enlightenment only slowly. A revolution may bring about the end of a personal despotism or of avaricious tyrannical oppression, but never a true reform of modes of thought. New prejudices will serve, in place of the old, as guide lines for the unthinking multitude. This enlightenment requires nothing but freedom— and the most innocent of all that may be called “freedom”: freedom to make public use of one’s reason in all matters. Now I hear the cry from all sides: “Do not argue!” The officer says: “Do not argue—drill!” The tax collector: “Do not argue—pay!” The pastor: “Do not argue—believe!” Only one ruler in the world says: “Argue as much as you please, but obey!” We find restrictions on freedom everywhere. But which restriction is harmful to enlightenment? Which restriction is innocent, and which advances enlightenment? I reply: the public use of one’s reason must be free at all times, and this alone can bring enlightenment to mankind.
Q. Which of the following best captures the main idea of the passage?
  • a)
    To highlight the perils of freedom without enlightenment
  • b)
    To trace the path from nonage to freedom
  • c)
    To evaluate the attainment of enlightenment
  • d)
    To elucidate the factors behind Enlightenment
Correct answer is option 'D'. Can you explain this answer?

Aditya Kumar answered
The author, in the passage, takes an explanatory approach. S/he defines the various reasons that are holding the public back from achieving enlightenment. S/he also analyses the prices to be paid for freedom without enlightenment. Hence, option 4 is the correct answer. Option 1 is too narrow. Option 2 is wrong as the author doesn’t focus on freedom. Option 3 is distorted. The author can’t evaluate something which has not been attained.

In public Greek life, a man had to make his way at every step through the immediate persuasion of the spoken word. Whether it be addressing an assembly, a law-court or a more restricted body, his oratory would be a public affair rather than under the purview of a quiet committee, without the support of circulated commentary, and with no backcloth of daily reportage to make his own or others‘ views familiar to his hearers. The oratory's immediate effect was all-important; it would be naive to expect that mere reasonableness or an inherently good case would equate to a satisfactory appeal. Therefore, it was early realized that persuasion was an art, up to a point teachable, and a variety of specific pedagogy was well established in the second half of the fifth century. When the sophists claimed to teach their pupils how to succeed in public life, rhetoric was a large part of what they meant, though, to do them justice, it was not the whole.
Skill naturally bred mistrust. If a man of good will had need of expression advanced of mere twaddle, to learn how to expound his contention effectively, the truculent or pugnacious could be taught to dress their case in well-seeming guise. It was a standing charge against the sophists that they ‗made the worse appear the better cause,‘ and it was this immoral lesson which the hero of Aristophanes‘ Clouds went to learn from, of all people, Socrates. Again, the charge is often made in court that the opponent is an adroit orator and the jury must be circumspect so as not to let him delude them. From the frequency with which this crops up, it is patent that the accusation of cleverness might damage a man. In Greece, juries, of course, were familiar with the style, and would recognize the more evident artifices, but it was worth a litigant‘s while to get his speech written for him by an expert. Persuasive oratory was certainly one of the pressures that would be effective in an Athenian law-court.
A more insidious danger was the inevitable desire to display this art as an art. It is not easy to define the point at which a legitimate concern with style shades off into preoccupation with manner at the expense of matter, but it is easy to perceive that many Greek writers of the fourth and later centuries passed that danger point. The most influential was Isocrates, who polished for long years his pamphlets, written in the form of speeches, and taught to many pupils the smooth and easy periods he had perfected. Isocrates took to the written word in compensation for his inadequacy in live oratory; the tough and nervous tones of a Demosthenes were far removed from his, though they, too, were based on study and practice. The exaltation of virtuosity did palpable harm. The balance was always delicate, between style as a vehicle and style as an end in itself.
 We must not try to pinpoint a specific moment when it, once and for all, tipped over; but certainly, as time went on, virtuosity weighed heavier. While Greek freedom lasted, and it mattered what course of action a Greek city decided to take, rhetoric was a necessary preparation for public life, whatever its side effects. It had been a source of strength for Greek civilization that its problems, of all kinds, were thrashed out very much in public. The shallowness which the study of rhetoric might (not must) encourage was the corresponding weakness.  
Directions: Read the above paragraph and answer the following
Q.If the author of the passage travelled to a political convention and saw various candidates speak he would most likely have the highest regard for an orator who:  
  • a)
    roused his hearers to immediate and decisive action.  
  • b)
    understood that rhetoric serves an aesthetic as well as a practical purpose.  
  • c)
    relied on facts and reason rather than on rhetorical devices in making his case.  
  • d)
    passed on the techniques he had perfected to many students.  
  • e)
    made use of flowery and inflated words 
Correct answer is option 'C'. Can you explain this answer?

Mapping the Passage:
¶1 introduces the importance of rhetoric in Greek life and the fact that it was taught.
¶2 explains why rhetorical skill was sometimes mistrusted, but still sought after.
¶3 states that as rhetoric (in writing and speech) became more of an art, its original purpose was corrupted.
¶4 states that the Greek system required rhetorical skills and therefore inherited rhetoric's drawbacks as well.
What is the author‘s main argument about oratory? It was necessary for the Greeks, but became a "weakness" when they focused too much on making it artistic (¶s 3 and 4). Therefore the author would admire an orator who didn't sacrifice the facts and reason to too much rhetoric. (C) keeps the good parts of rhetoric while leaving out the artistic flourishes the author dislikes.

In all battles two things are usually required of the Commander-in-Chief: to make a good plan for his army and to keep a strong reserve. Both of these are also obligatory for the painter. To make a plan, thorough reconnaissance of the country where the battle is to be fought is needed. Its fields, its mountains, its rivers, its bridges, its trees, its flowers, its atmosphere—all require and repay attentive observation from a special point of view.
I think this is one of the chief delights that have come to me through painting. No doubt many people who are lovers of art have acquired it to a high degree without actually practicing. But I expect that nothing will make one observe more quickly or more thoroughly than having to face the difficulty of representing the thing observed. And mind you, if you do observe accurately and with refinement, and if you do record what you have seen with tolerable correspondence, the result follows on the canvas with startling obedience.
But in order to make his plan, the General must not only reconnoitre the battle-ground; he must also study the achievements of the great Captains of the past. He must bring the observations he has collected in the field into comparison with the treatment of similar incidents by famous chiefs.
Considering this fact, the galleries of Europe take on a new—and to me at least — a severely practical interest. You see the difficulty that baffled you yesterday; and you see how easily it has been overcome by a great or even by a skilful painter. Not only is your observation of Nature sensibly improved and developed, but also your comprehension of the masterpieces of art.
But it is in the use and withholding of their reserves that the great commanders have generally excelled. After all, when once the last reserve has been thrown in, the commander‘s part is played. If that does not win the battle, he has nothing else to give. Everything must be left to luck and to the fighting troops. But these last reserves, in the absence of high direction, are apt to get into sad confusion, all mixed together in a nasty mess, without order or plan—and consequently without effect.  
Mere masses count no more. The largest brush, the brightest colours cannot even make an impression. The pictorial battlefield becomes a sea of mud mercifully veiled by the fog of war. Even though the General plunges in himself and emerges bespattered, as he sometimes does, he will not retrieve the day. In painting, the reserves consist in Proportion or Relation. And it is here that the art of the painter marches along the road which is traversed by all the greatest harmonies in thought. At one side of the palette there is white, at the other black; and neither is ever used 'neat.‘ Between these two rigid limits all the action must lie, all the power required must be generated. Black and white themselves placed in juxtaposition make no great impression; and yet they are the most that you can do in pure contrast.  
Directions: Read the above paragraph and answer the following
Q.The author‘s statement ―"But [the fighting troops], in the absence of high direction, are apt to get into sad confusion, all mixed together in a nasty mess, without order or plan—and consequently without effect" assumes that
  • a)
    chaotic painting cannot have an unintended artistic effect.  
  • b)
    an artist naturally resists direction from another individual.  
  • c)
    a painting cannot help but reflect the mental state of its painter.  
  • d)
    it is impossible for painters to collaborate on a work without confusion.
  • e)
    troops always need someone to guide them 
Correct answer is option 'A'. Can you explain this answer?

Prerna Menon answered
Mapping the Passage
¶1 compares painting a picture to fighting a battle, lists two similarities, planning and
backup, and discusses planning.
¶2 explains that practicing art is a great way to become a lover of art.
¶s3 and 4 draws in the analogy of the general and explain the need to study previous masters in war and art.
¶s5 and 6 explain the need to keep reserves in battle and painting.
Yet another question testing your understanding of the author‘s extended metaphor. These will be very common in any passage where unusual parallels are drawn. The quoted statement comes from ¶5; since all of the answer choices mention painting, work through how this part of the metaphor corresponds. The author is arguing that without a reserve, colours, like troops, will be confused and without order and therefore useless. For this to be true, the author must also believe that a painting without order suffers artistically, choice (A). To test an assumption in your practice, use the denial test: If the author does in fact assume X, the argument should fall apart if X is false. In this case, if chaotic painting can have an artistic effect, then the author‘s point about confused troops becomes meaningless. The assumption as it is written is therefore valid.

Direction For question : As the Heart of Darkness makes its way into the savage shadows of the African continent, Joseph Conrad exposes a psycho-geography of the collective unconscious in the entangling symbolic realities of the serpentine Congo. Conrad’s novella descends into the unknowable darkness at the heart of Africa, taking its narrator, Marlow, on an underworld journey of individuation, a modern odyssey toward the center of the Self and the center of the Earth. Ego dissolves into soul as, in the interior; Marlow encounters his double in the powerful image of ivory-obsessed Kurtz, the dark shadow of European imperialism. The dark meditation is graced by personifications of anima in Kurtz’ black goddess, the savagely magnificent consort of the underworld, and in his porcelain -skinned Persephone, innocent intended of the upper world. “Africa,” wrote Graham Greene, “will always be the Africa in the Victorian atlas, the blank unexplored continent in the shape of the human heart.” The African heart described by Greene “acquired a new layer of meaning when Conrad portrayed the Congo under King Leopold as the Heart of Darkness, a place where barbarism triumphs over humanity, nature over technology, biology over culture, id over super ego.” The unknown and uncharted topography of the African continent first beckoned Conrad’s narrator, Marlow, into its depths in his boyhood: “Now, when I was a little chap I had a passion for maps. I would look for hours at South America, or Africa, or Australia, and lose myself in all the glories of exploration”. When Marlow was grown and Africa was no longer a blank space on the map, but rather “a place of darkness,” there was still one river there that drew him especially, “a mighty big river, that you could see on the map, resembling an immense snake uncoiled, with its head in the sea, its body at rest curving afar over a vast country, and its tail lost in the depths of the land”. This same deep place, the Congo, that had seduced Conrad’s ivory hunting Kurtz into the horrors of its savage embrace had, in 1890, lured Conrad himself into adventure that turned him from sailor to writer and severely affected his health for the rest of his life. As the voyage up the Congo pro ved fateful for the development of Conrad’s narrator, Marlow, it was equally fateful for Conrad’s individuation, as he reflects in his letters “Before the Congo I was just a mere animal.” Hillman, in “Notes on White Supremacy” reminds us that, like Conrad, both Freud and Jung were called to venture into the shadowed continent and vestiges of their journeys still colour our psychological language: The convention informing geographical discoveries and the expansion of white consciousness over Africa continue to inform psychic geography. The topological language used by Freud for “the unconscious” as a place below, different, timeless, primordial, libidinal and separated from the consciousness recapitulates what white reporters centuries earlier said about West Africa. From Conrad’s Heart of Darkness to van der Post’s Venture to the Interior, Africa and the unconscious allegorize the other place.... “Just don’t stay in the topical colonies too long; you must reign at home,” writes Freud in 1911 to Jung, who himself made the African journey fourteen years later, describing the vast lands and dark peoples he encountered in language he applies as well to the immemorial unconscious psyche.... Part of psychology’s myth is that the unconscious was “discovered” as its contents are “explored”. Thus Africa has become a topology of the mind — its location, its shape, its cultures, its textures, its rhythms, its foliage, its hues, its wildness — all calling forth something lost in the psychology of the white European. It is with an understanding of our destiny to explore that symbolic lost continent within ourselves that we can begin to appreciate the prescience of Jungian psychology in Conrad’s Heart of Darkness.
Q. Identify the nature of author’s tone in the above passage.
  • a)
    Simply Caustic
  • b)
    Sincerely Idealistic
  • c)
    Descriptive
  • d)
    Argumentative
Correct answer is option 'C'. Can you explain this answer?

Ankita Yadav answered
Descriptive
The tone of the passage is descriptive as the author provides a detailed analysis of Joseph Conrad's novella, "Heart of Darkness," and its exploration of the African continent. The author delves into the symbolic realities and psychological depths of the story, highlighting key themes such as individuation, imperialism, and the unconscious. The passage also discusses the personal experiences of Conrad and his narrator, Marlow, in relation to Africa, drawing parallels between the physical journey into the heart of the continent and the psychological journey into the depths of the mind.

Key Points:
- Detailed analysis of Conrad's novella
- Exploration of themes such as individuation and imperialism
- Comparison between physical and psychological journeys
- Personal experiences of Conrad and Marlow
- Parallels drawn between Africa and the unconscious

Direction For question : As the Heart of Darkness makes its way into the savage shadows of the African continent, Joseph Conrad exposes a psycho-geography of the collective unconscious in the entangling symbolic realities of the serpentine Congo. Conrad’s novella descends into the unknowable darkness at the heart of Africa, taking its narrator, Marlow, on an underworld journey of individuation, a modern odyssey toward the center of the Self and the center of the Earth. Ego dissolves into soul as, in the interior; Marlow encounters his double in the powerful image of ivory-obsessed Kurtz, the dark shadow of European imperialism. The dark meditation is graced by personifications of anima in Kurtz’ black goddess, the savagely magnificent consort of the underworld, and in his porcelain -skinned Persephone, innocent intended of the upper world. “Africa,” wrote Graham Greene, “will always be the Africa in the Victorian atlas, the blank unexplored continent in the shape of the human heart.” The African heart described by Greene “acquired a new layer of meaning when Conrad portrayed the Congo under King Leopold as the Heart of Darkness, a place where barbarism triumphs over humanity, nature over technology, biology over culture, id over super ego.” The unknown and uncharted topography of the African continent first beckoned Conrad’s narrator, Marlow, into its depths in his boyhood: “Now, when I was a little chap I had a passion for maps. I would look for hours at South America, or Africa, or Australia, and lose myself in all the glories of exploration”. When Marlow was grown and Africa was no longer a blank space on the map, but rather “a place of darkness,” there was still one river there that drew him especially, “a mighty big river, that you could see on the map, resembling an immense snake uncoiled, with its head in the sea, its body at rest curving afar over a vast country, and its tail lost in the depths of the land”. This same deep place, the Congo, that had seduced Conrad’s ivory hunting Kurtz into the horrors of its savage embrace had, in 1890, lured Conrad himself into adventure that turned him from sailor to writer and severely affected his health for the rest of his life. As the voyage up the Congo pro ved fateful for the development of Conrad’s narrator, Marlow, it was equally fateful for Conrad’s individuation, as he reflects in his letters “Before the Congo I was just a mere animal.” Hillman, in “Notes on White Supremacy” reminds us that, like Conrad, both Freud and Jung were called to venture into the shadowed continent and vestiges of their journeys still colour our psychological language: The convention informing geographical discoveries and the expansion of white consciousness over Africa continue to inform psychic geography. The topological language used by Freud for “the unconscious” as a place below, different, timeless, primordial, libidinal and separated from the consciousness recapitulates what white reporters centuries earlier said about West Africa. From Conrad’s Heart of Darkness to van der Post’s Venture to the Interior, Africa and the unconscious allegorize the other place.... “Just don’t stay in the topical colonies too long; you must reign at home,” writes Freud in 1911 to Jung, who himself made the African journey fourteen years later, describing the vast lands and dark peoples he encountered in language he applies as well to the immemorial unconscious psyche.... Part of psychology’s myth is that the unconscious was “discovered” as its contents are “explored”. Thus Africa has become a topology of the mind — its location, its shape, its cultures, its textures, its rhythms, its foliage, its hues, its wildness — all calling forth something lost in the psychology of the white European. It is with an understanding of our destiny to explore that symbolic lost continent within ourselves that we can begin to appreciate the prescience of Jungian psychology in Conrad’s Heart of Darkness.
Q. According to the passage, which of the following is not true about The Congo?
  • a)
    Congo was responsible for the deteriorating health of Conrad.
  • b)
    Congo was a powerful force that enabled the change of profession of people.
  • c)
    Congo’s power is compared to the devouring powers of a snake.
  • d)
    Congo proved itself fatalistic for different characters in the novel.
Correct answer is option 'D'. Can you explain this answer?

Ankita Yadav answered
Incorrect Statement:

Congo proved itself fatalistic for different characters in the novel.
---

Explanation:
- The statement that Congo proved itself fatalistic for different characters in the novel is not true based on the information provided in the passage.
- The passage mentions that the Congo was a place that drew individuals like Marlow and Kurtz into its depths, leading to profound psychological and personal transformations.
- While the Congo did have a significant impact on the health of Conrad, it did not prove fatalistic for different characters in the novel.
- Instead, the Congo symbolizes a complex and mysterious force that triggers deep introspection and individuation in those who venture into its depths.
- Therefore, the idea that the Congo was fatalistic for various characters in the novel is not supported by the passage.

Direction For Question : Enlightenment is man’s emergence from his self- imposed nonage. Nonage is the inability to use one’s own understanding without another’s guidance. This nonage is self-imposed if its cause lies not in lack of understanding but in indecision and lack of courage to use one’s own mind without another’s guidance. Dare to know! (Sapere aude.) “Have the courage to use your own understanding,” is therefore the motto of the enlightenment. Laziness and cowardice are the reasons why such a large part of mankind gladly remain minors all their lives, long after nature has freed them from external guidance. They are the reasons why it is so easy for others to set themselves up as guardians. It is so comfortable to be a minor. If I have a book that thinks for me, a pastor who acts as my conscience, a physician who prescribes my diet, and so on—then I have no need to exert myself. I have no need to think, if only I can pay; others will take care of that disagreeable business for me. Thus it is very difficult for the individual to work himself out of the nonage which has become almost second nature to him. He has even grown to like it, and is at first really incapable of using his own understanding because he has never been permitted to try it. Dogmas and formulas, these mechanical tools designed for reasonable use—or rather abuse— of his natural gifts, are the fetters of an everlasting nonage. The man who casts them off would make an uncertain leap over the narrowest ditch, because he is not used to such free movement. That is why there are only a few men who walk firmly, and who have emerged from nonage by cultivating their own minds. It is more nearly possible, however, for the public to enlighten itself; indeed, if it is only given freedom, enlightenment is almost inevitable. There will always be a few independent thinkers, even among the self- appointed guardians of the multitude. Once such men have thrown off the yoke of nonage, they will spread about them the spirit of a reasonable appreciation of man’s value and of his duty to think for himself. It is especially to be noted that the public which was earlier brought under the yoke by these men, afterwards forces these very guardians to remain in submission, if it is so incited by some of its guardians who are themselves incapable of any enlightenment. That shows how pernicious it is to implant prejudices: they will eventually revenge themselves upon their authors or their authors’ descendants. Therefore, a public can achieve enlightenment only slowly. A revolution may bring about the end of a personal despotism or of avaricious tyrannical oppression, but never a true reform of modes of thought. New prejudices will serve, in place of the old, as guide lines for the unthinking multitude. This enlightenment requires nothing but freedom— and the most innocent of all that may be called “freedom”: freedom to make public use of one’s reason in all matters. Now I hear the cry from all sides: “Do not argue!” The officer says: “Do not argue—drill!” The tax collector: “Do not argue—pay!” The pastor: “Do not argue—believe!” Only one ruler in the world says: “Argue as much as you please, but obey!” We find restrictions on freedom everywhere. But which restriction is harmful to enlightenment? Which restriction is innocent, and which advances enlightenment? I reply: the public use of one’s reason must be free at all times, and this alone can bring enlightenment to mankind.
Q. What is the author trying to instruct us through this passage?
  • a)
    S/he wants us to think freely and act judiciously and wisely.
  • b)
    S/he is promoting the tolerance of diverse viewpoints attain freedom.
  • c)
    S/he is instructing us that while expressing our opinions, we should not forget to discharge our duties.
  • d)
    S/he encourages us to be free in mind and spirit at all cost.
Correct answer is option 'A'. Can you explain this answer?

Janhavi Reddy answered
Analysis of the Passage:

Encouraging Free Thinking:
The author is emphasizing the importance of using one's own understanding and thinking freely without relying on others for guidance. This is evident from the quote "Have the courage to use your own understanding," which is highlighted as the motto of enlightenment in the passage.

Promoting Wise and Judicious Action:
The author is instructing readers to act judiciously and wisely by using their own reasoning abilities. This is indicated by the author's criticism of laziness and cowardice, which prevent individuals from thinking for themselves.

Importance of Independence in Thought:
The passage underscores the significance of individuals cultivating their own minds and breaking free from the constraints of dogmas and formulas imposed by others. This independence in thought is crucial for personal growth and enlightenment.

Encouraging Freedom of Reasoning:
The author advocates for the freedom to make public use of one's reason in all matters as a means to achieve enlightenment. This freedom of reasoning is portrayed as essential for the progress of mankind.
In conclusion, the author's main instruction through this passage is to encourage individuals to think freely, act judiciously, and use their own understanding to attain enlightenment.

Direction For Question : Enlightenment is man’s emergence from his self- imposed nonage. Nonage is the inability to use one’s own understanding without another’s guidance. This nonage is self-imposed if its cause lies not in lack of understanding but in indecision and lack of courage to use one’s own mind without another’s guidance. Dare to know! (Sapere aude.) “Have the courage to use your own understanding,” is therefore the motto of the enlightenment. Laziness and cowardice are the reasons why such a large part of mankind gladly remain minors all their lives, long after nature has freed them from external guidance. They are the reasons why it is so easy for others to set themselves up as guardians. It is so comfortable to be a minor. If I have a book that thinks for me, a pastor who acts as my conscience, a physician who prescribes my diet, and so on—then I have no need to exert myself. I have no need to think, if only I can pay; others will take care of that disagreeable business for me. Thus it is very difficult for the individual to work himself out of the nonage which has become almost second nature to him. He has even grown to like it, and is at first really incapable of using his own understanding because he has never been permitted to try it. Dogmas and formulas, these mechanical tools designed for reasonable use—or rather abuse— of his natural gifts, are the fetters of an everlasting nonage. The man who casts them off would make an uncertain leap over the narrowest ditch, because he is not used to such free movement. That is why there are only a few men who walk firmly, and who have emerged from nonage by cultivating their own minds. It is more nearly possible, however, for the public to enlighten itself; indeed, if it is only given freedom, enlightenment is almost inevitable. There will always be a few independent thinkers, even among the self- appointed guardians of the multitude. Once such men have thrown off the yoke of nonage, they will spread about them the spirit of a reasonable appreciation of man’s value and of his duty to think for himself. It is especially to be noted that the public which was earlier brought under the yoke by these men, afterwards forces these very guardians to remain in submission, if it is so incited by some of its guardians who are themselves incapable of any enlightenment. That shows how pernicious it is to implant prejudices: they will eventually revenge themselves upon their authors or their authors’ descendants. Therefore, a public can achieve enlightenment only slowly. A revolution may bring about the end of a personal despotism or of avaricious tyrannical oppression, but never a true reform of modes of thought. New prejudices will serve, in place of the old, as guide lines for the unthinking multitude. This enlightenment requires nothing but freedom— and the most innocent of all that may be called “freedom”: freedom to make public use of one’s reason in all matters. Now I hear the cry from all sides: “Do not argue!” The officer says: “Do not argue—drill!” The tax collector: “Do not argue—pay!” The pastor: “Do not argue—believe!” Only one ruler in the world says: “Argue as much as you please, but obey!” We find restrictions on freedom everywhere. But which restriction is harmful to enlightenment? Which restriction is innocent, and which advances enlightenment? I reply: the public use of one’s reason must be free at all times, and this alone can bring enlightenment to mankind.
Q. Which of the following is closest to being an example of ‘self-imposed nonage’?
  • a)
    A young boy refusing to do his homework without the guidance of his mother
  • b)
    A teenage girl not obeying her father’s order to return home by 7 in the evening
  • c)
    A cricketer not using an aggressive batting style during a run chase as he is waiting for his coach’s permission
  • d)
    A senior employee refusing to follow the guidelines to improve productivity set by the upper management due to creative Differences
Correct answer is option 'C'. Can you explain this answer?

Om Mukherjee answered
Refer to the lines – “Nonage is the inability to use one’s own...indecision and lack of courage to use one’s own mind without another’s guidance.” There are two components here: a – inability to use one’s knowledge without guidance; b – no lack of understanding. Only option 3 clearly shows the two things. The batsman knows how to bat aggressively; he is simply indecisive. In 1, the boy might not know the answer to questions of the homework. 2 is irrelevant. 4 has not component of indecisiveness. The employee is quite decisive.

By regarding the expanding universe as a motion picture, you can easily imagine ―running the film backward.‖ If you do so, you find the universe getting smaller and smaller, and eventually you come to the moment when its whole mass is crammed into an infinitely dense point. Before that time it didn‘t exist, or at least it didn‘t exist in its present form. Though there is some controversy about its exact age, most cosmologists would be inclined to agree that the universe has existed for about ten to twenty billion years. For scale, this can be compared to the four-and-a-half-billion-year age of the solar system, the time since the disappearance of the dinosaurs (sixty-five million years), and the age of the human race (about three million years).  
The event that marked the beginning of the universe was christened the Big Bang; the term has now entered the vernacular of our culture. Originally the name referred only to the single initiating event; now, however, astronomers have come to use it to mean the entire developmental process of the birth and expansion of the cosmos.  
The simple statement that the universe had a beginning in time is by now so obvious to astrophysicists that few give it a second thought. Yet it is a statement that has profound implications. Most civilizations embrace one of two opposite concepts of time. Linear time has a beginning, a duration, and an end; cyclical time, as its name suggests, continues around and around forever. In a universe that functions through cyclical time, the question of creation never arises; the universe always was and always will be. The minute you switch to linear time you immediately confront the vexing question not only of creation, but also of the Creator. Although there is no logical reason for the assumption, many people believe that if something comes into existence, it must do so in response to the actions of some rational being. Because of that belief, astronomers, even though they resist becoming involved in theological discussion, find themselves in one when they posit the Big Bang universe. It puts them squarely in the middle of an age-old debate.  
One common misconception about the Big Bang that should be disposed of immediately is the notion that the universal expansion is analogous to the explosion of an artillery shell. The galaxies are not like bits of shrapnel speeding away from a central explosion. The raisin-indough analogy is a more satisfactory way to think about the whole process.  
Directions: Read the above paragraph and answer the following:
Q.According to the passage, which of the following statements is NOT true?  
  • a)
    Many people believe that a rational impetus created the universe.  
  • b)
    The solar system was created immediately after the Big Bang.  
  • c)
    The universe is larger today than it was in the past.  
  • d)
    Different societies measure time differently.                  
  • e)
    Most cosmologists believe the universe to be 10 to 20 billion years old 
Correct answer is option 'B'. Can you explain this answer?

Parijat Menon answered
Explanation:

Incorrect Statement:
- The statement "The solar system was created immediately after the Big Bang" is not true according to the passage.

Reasoning:
- The passage mentions that the universe has existed for about ten to twenty billion years, while the solar system is about four-and-a-half-billion years old. This age gap indicates that the solar system was not created immediately after the Big Bang.
- The passage also discusses the concept of linear time and the beginning of the universe with the Big Bang event, which led to the expansion of the cosmos. This expansion does not imply that the solar system was created at the same time as the Big Bang.
Therefore, based on the information provided in the passage, the statement that the solar system was created immediately after the Big Bang is not true.

As formal organizations, business corporations are distinguished by their particular goals, which include maximization of profits, growth, and survival. Providing goods and services is a means to this end. If, for example, a number of individuals (outsiders or even insiders) believe that a company‘s aggressive marketing of infant formula in third world countries is morally wrong, the company is unlikely to be moved by arguments based on ethos alone as long as what it is doing remains profitable. But if those opposed to the company‘s practice organize a highly effective boycott of the company‘s products, their moral views will soon enter into the company‘s deliberations indirectly as limiting operating conditions. They can, at this point, no more be ignored than a prohibitive increase in the costs of certain raw materials.  
Although the concepts and categories of ethics may be applied to the conduct of corporations, there are important differences between the values and principles underlying corporate behaviour and those underlying the actions of most individuals. If corporations are by their nature end- or goal-directed how can they acknowledge acts as wrong in and of themselves? Is it possible to hold one criminally responsible for acts that if performed by a human person would result in criminal liability?  
The first case of this type to achieve widespread public attention was the attempt to prosecute the Ford Motor Company for manslaughter as the result of alleged negligent or reckless decision making concerning the safety engineering of the Pinto vehicle. Although the defendant corporation and its officers were found innocent after trial, the case can serve as an exemplar for our purposes.  
In essence, the prosecution in this case attempted to show that the corporation had produced and distributed a vehicle that was known to be defective at the time of production and sale, and that even after a great deal of additional information accumulated regarding the nature of the problems, the corporation took no action to correct them. The obvious non-corporate analogy would be the prosecution of a person who was driving a car with brakes known to be faulty, who does not have them repaired because it would cost too much, and who kills someone when the brakes eventually fail and the car does not stop in time. Such cases involving individuals are prosecuted and won regularly.
If corporations have no concept of right or wrong because they are exclusively goal-directed, can they be convicted in cases of this type, and what purpose would be served by such a conviction? Perhaps we can make a utilitarian argument for convicting corporations of such crimes. The argument would be that of deterrence; conviction and punishment would deter other corporations from taking similar actions under similar circumstances. However, there appears to be considerable evidence that deterrence does not work on corporations, even if, arguably, it works on individuals. The possibility of being discovered and the potential magnitude of the fine merely become more data to be included in the analysis of limiting conditions.  
Directions: Read the above paragraph and answer the following:
Q. If a company that produced shampoo products opted to stop the routine testing of its products on animals because it decided that it is wrong to cause the animals pain, what effect would this have on the argument made in the passage?  
  • a)
    It would strongly support the argument.  
  • b)
    It would substantially weaken the argument.  
  • c)
    It would neither support nor substantially weaken the argument.  
  • d)
    It would support the argument somewhat, but not conclusively.  
  • e)
    It would weaken the argument only if the company is a government owned company 
Correct answer is option 'D'. Can you explain this answer?

Rishika Sen answered
The correct answer is D. It would support the argument somewhat, but not conclusively.
The argument in the passage is that corporations are primarily goal-directed and focused on maximizing profits, growth, and survival. They are not inherently concerned with ethical considerations unless it directly affects their profitability.

If a company decides to stop routine animal testing because it believes it is wrong to cause animals pain, it does show that the company is considering ethical concerns. However, this decision alone does not conclusively prove that corporations can acknowledge acts as wrong in and of themselves. It could be argued that the company made this decision because it believes it will enhance its reputation and attract more customers who are concerned about animal welfare. Therefore, while it does support the argument to some extent, it does not provide conclusive evidence.

The original Hellenistic community was idealized, the Greeks‘ own golden dream—a community never achieved but only imagined by the Macedonian Alexander, who was possessed of the true faith of all converts to a larger vision. The evolving system of city-states had produced not only unity with a healthy diversity but also narrow rivalries. No Hellenic empire arose, only scores of squabbling cities pursuing bitter feuds born of ancient wrongs and existing ambitions. It was civil strife made possible by isolation from the great armies and ambitions of Asia.  
Greek history could arguably begin in July of 776 B.C., the First Olympiad, and end with Theodosus‘s ban on the games in 393 A.D. Before this there had been a long era of two tribes, the Dorians and Ionians, scarcely distinguishable to the alien eye, but distinctly separate in their own eyes until 776. After Theodosus' ban most of the Mediterranean world was Greek-like, in fact, but the central core had been rendered impotent by diffusion.  
During the eventful Greek millennium, the Olympics reflected not the high ideals of Hellenes but rather the mean reality of the times. Its founders had created a monster, games that twisted the strategists‘ aspirations to unity to fit the unpleasant reality of the Hellenistic world. The games not only mirrored the central practices of the Greek world that reformers would deny but also imposed the flaws of that world. Like the atomic theory of the Greek philosophers, the Greek gamers‘ theories were far removed from reality; they were elegant, consistent, logical, and irrelevant.
Part religious ritual, part game rite, in the five-day Olympic Games, various athletes coming together under the banner of their cities; winning became paramount, imposing defeat a delight. As Greek society evolved, so, too, did the games, but rarely as a unifying force. Athletes supposedly competing for the laurel of accomplishment in the name of idealism found that dried olive leaves changed to gold. Each local polis (city-state) sought not to contribute to the grandeur of Greece, but to achieve its own glory. As in the real world, in the games no Greek could trust another, and each envied rivals' victories. The Olympic spirit was not one of communal bliss but bitter lasting competition institutionalized in games. 
Directions: Read the above paragraph and answer the following
Q. For which of the following statements does the passage provide some evidence or explanation?  
I. Alexander united ancient Greece through a series of military conquests.  
II. The divisions among Greek city-states were reflected in the Olympics.  
III. The Olympic Games could not have occurred without a city-state system. 
  • a)
    II only  
  • b)
    III only  
  • c)
    I and II  
  • d)
    II and III  
  • e)
    I, II and III  
Correct answer is option 'A'. Can you explain this answer?

Mapping the Passage
¶1 argues that in reality the Hellenic period was tumultuous, not the idealized community that Alexander desired.
¶2 gives a time frame for Greek civilization and the Olympic games.
¶3 argues that the games reflected Greek culture, but not positively as the founders intended.
¶4 argues that the games reinforced disunity instead of promoting the unity originally intended.
Take a moment to remind yourself of the author‘s main point about the Games and look at the layout of the choices before trying to answer. RN II is the most frequent, so hit that first. RN II is basically the author‘s main argument, and the passage itself is explanation and example for this. Eliminate (B). RN I offers a point not made by the passage: the author argues that Alexander never truly unified Greece (and he offers no evidence for this). Eliminate (C). The author never makes the claim in RN III, and therefore (D) can be eliminated. (A) alone is left.

While many points are worth making in an evaluation of the single sixyear presidential term, one of the most telling points against the single term has not been advanced. This kind of constitutional limitation on elections is generally a product of systems with weak or non-existent political parties.
 Since there is no party continuity or corporate party integrity in such systems, there is no basis for putting trust in the desire for re-election as a safeguard against mismanagement in the executive branch. Better under those conditions to operate on the basis of negative assumptions against incumbents. I do not know if the earliest proposal for a single, nonrepeatable term was made in the 1820s because that was a period of severely weak political parties. But I do feel confident that this is a major reason, if not the only reason, that such a proposal has been popular since the 1940s.
 Though the association of the non-repeatable election with weak political parties is not in itself an argument against the limitation, the fallout from this association does contribute significantly to the negative argument. Single-term limitations are strongly associated with corruption. In any weak party system, including the presidential system, the onus of making deals and compromises, both shady and honourable, rests heavily upon individual candidates. Without some semblance of corporate integrity in a party, individual candidates have few opportunities to amortize their obligations across the spectrum of elective and appointive jobs and policy proposals.
The deals tend to be personalized and the payoffs come home to roost accordingly.  If that situation is already endemic in conditions of weak or nonexistent parties, adding to it the limitation against re-election means that candidates and officials, already prevented from amortizing their deals across space, are also unable to amortize their obligations temporally. This makes for a highly beleaguered situation. The single six-year term for presidents is an effort to compensate for the absence of a viable party system, but it is a compensation ultimately paid for by further weakening the party system itself.
 Observers, especially foreign observers, have often noted that one source of weakness in American political parties is the certainty of election every two or four years, not only because any artificial limitation on elections is a violation of democratic principles but also because when elections are set in a certain and unchangeable cycle, political parties do not have to remain alert but can disappear into inactivity until a known point prior to the next election. To rigidify matters by going beyond the determinacy of the electoral cycle to add an absolute rule of one term would hang still another millstone around the neck of already doddering political parties.  
Directions: Read the above paragraph and answer the following:
According to the passage, which of the following is most likely to be true of a political system with weak political parties? 
  • a)
    Politicians appoint unqualified people to important posts.  
  • b)
    Political parties favour frequent elections.  
  • c)
    Political bargains are made by individual candidates.  
  • d)
    Elections tend to occur with very great frequency.  
  • e)
    It encourages politicians to be more honest 
Correct answer is option 'C'. Can you explain this answer?

Ananya Sharma answered
Mapping the Passage
¶1 introduces the idea of a single presidential term.
¶2 argues that the single term is usually associated with countries with weak political parties and therefore popular when political parties are weak.
¶s3 and 4 argue that single-term systems encourage corruption.
¶5 argues that the single-term system is designed to make up for weak parties, but ends up making parties even weaker.
What does the author say about weak political parties? They lead to a preference for single terms, which, when enacted, lead to corruption. Evaluate the choices with this main chain of causes and effects in mind. While three choices aren‘t touched on by the author, (C) is the main point made in ¶3 and the beginning of ¶ 4: single-term systems encourage individual bargains (which ultimately lead to corruption.)

Measuring more than five feet tall and ten feet long, the Javan rhinoceros is often called the rarest large mammal on earth. None exist in zoos. Like the Indian rhino, the Javan has only one horn; African and Sumatran rhinos have two. While the Javan rhino habitat once extended across southern Asia, now there are fewer than one hundred of the animals in Indonesia and under a dozen in Vietnam. Very little is known about Javan rhinos because they lead secretive and solitary lives in remote jungles.
Until recently, scientists debated whether females even have horns, and most scientific work has had to rely on DNA garnered from dung.
The near extinction of the Javan rhino is the direct result of human actions. For centuries, farmers, who favored the same habitat, viewed them as crop eating pests and shot them on sight. During the colonial period, hunters slaughtered thousands. Now, human efforts to save them may well prove futile. The Vietnamese herd is probably doomed, as too few remain to maintain the necessary genetic variation. Rhinos from Java cannot supplement the Vietnamese numbers because in the millions of years since Indonesia separated from the mainland, the two groups have evolved into separate sub-species. In Indonesia, the rhinos are protected on the Ujung Kulon peninsula, which is unsettled by humans, and still have sufficient genetic diversity to have a chance at survival.
Ironically, however, the lack of human disturbance allows mature forests to replace the shrubby vegetation the animals prefer. Thus, human benevolence may prove little better for these rhinos than past human maltreatment.
Q.
The purpose of the first paragraph is to
  • a)
    discuss the different types of rhinoceroses that populate the world
  • b)
    describe the ways in which human actions have brought the Javan rhino close to extinction
  • c)
    outline the few known facts about the Javan rhino
  • d)
    discuss the steps taken to save the Javan rhino
  • e)
    highlight the differences between the sub-species of Javan rhinos in Vietnam and Indonesia
Correct answer is option 'C'. Can you explain this answer?

Saumya Shah answered
The first paragraph describes the Javan rhino and indicates how little is known about it. This should be reflected in the answer. It is the second paragraph that relates the effects of human activity on the rhinos and their chances for survival.
(A) This choice is too broad, as the paragraph focuses on the Javan rhino and only mentions its differences from some other rhinos to distinguish the species from others.  
(B) This answer choice describes the topic of the second paragraph, not the first.
(C) CORRECT. The first paragraph provides the known facts about the Javan rhino and indicates that much remains unknown.
(D) The steps taken to save the Javan rhino are only mentioned in the second paragraph; they do not appear in the first paragraph.
(E) The first paragraph only indicates the respective number of rhinos in Indonesia and Vietnam in passing, and does not mention the differences between the two groups. The fact that the two have evolved into separate sub-species is mentioned only in the second paragraph.

Film scholars agree that Hollywood portrayals of America at war follow a cyclical pattern. During and immediately after a conflict, important films trumpet glory and sacrifice. Ten to fifteen years later, questioning and sometimes pacifistic movies about the conflict dominate. In the late 1960’s, “the raging bulls” of Hollywood—the young trendsetters rising to prominence—proclaimed this pattern obsolete. However, the passage of time has demonstrated this cultural pattern to be more resilient than it seemed in those days of social change.
Throughout the majority of the last century, evidence of the cyclical portrayal of war in film abounds. After America declared war against Germany during World War I, the still infant film industry glorified the fight against “the Hun.” By the early 1930’s, major releases had changed their tone; for example, All Quiet on the Western Front put forth an anti-war message by displaying the horrors of combat. After World War II began, the industry shifted gears. Suddenly, important pictures again portrayed glories and courage without the questioning or despair. For example, Guadalcanal Diary, produced during the war, showed “the ultimate sacrifice” as a noble and undoubted good. Once again, though, by 1957, films such as The Bridge on the River Kwai won awards for depicting the moral confusion of war.
Those who later declared this pattern dead based their conviction on their hearts rather than their minds. During the Vietnam War, the only major film about that conflict was The Green Berets, starring John Wayne and far closer in tone to Guadalcanal Diary than to The Bridge on the River Kwai. Similarly, years went by before more complex visions of war, such as Apocalypse Now, and then Platoon, emerged.
While today’s film industry is more diverse and its audience more culturally fragmented, this cycle largely continues. Jarhead, a layered depiction of the first gulf war, premiered more than ten years after that conflict. Further evidence of this pattern can be seen in the release of Apocalypse Now Redux, which contained additional footage that the producers originally thought would repel audiences. Thus, the famous aphorism “The more things change, the more they stay the same” certainly applies to this aspect of the film industry.
Q.
According to the passage, Apocalypse Now Redux  differed from Apocalypse Now in which of the following ways?
  • a)
    The added footage made it less appealing to a moreculturally diverse audience.
  • b)
    The added footage made its portrayal of war lessglorified and more ambiguous.
  • c)
    The added footage made its portrayal of war lessharsh and more glorified.
  • d)
    The added footage made it more similar in tone toother war movies.
  • e)
    The removed footage made its portrayal of war lessglorified and less appealing.
Correct answer is option 'B'. Can you explain this answer?

Rhea Gupta answered
Passage 1
1. Apocalypse Now Redux and Apocalypse Now were discussed in the final two paragraphs. In the third paragraph, Apocalypse Now is described as a film that was released years after the conflict it portrayed and had a more complex view of the war. In the last paragraph, Apocalypse Now Redux is presented as further evidence that the pattern discussed in this passage continues. That pattern is that war movies presented less glorified and more layered portrayals when the conflict was further in the past. Thus, the correct answer will note that its perspective was more complex and morally ambiguous.

(A) This choice distorts the meaning of the passage. While the last paragraph notes that film audiences are more diverse, this is not connected to the information provided about Apocalypse Now Redux.
(B) CORRECT. The last  aragraph notes that the extra footage was not orginally included because it might repel audiences. Thus, the updated film's portrayal of war must be less appealing and more ambiguous than that of the  original.
(C) This choice indicates a change in the opposite direction; the last paragraph indicates that the additional footage made the film's perspective on war harsher as opposed to more glorified.
(D) The passage indicates that not all war movies had the same tone, and the answer choice does not specify which "other movies" are referred to; therefore, this
choice is incorrect.
(E) There is no mention of any removed footage in the passage. As it is not possible to know if any footage was removed, this choice is incorrect.

The golden toad of Costa Rica, whose beauty and rarity inspired an unusual degree of human interest from a public generally unconcerned about amphibians, may have been driven to extinction by human activity nevertheless. In the United States, a public relations campaign featuring the toad raised money to purchase and protect the toad’s habitat in Costa Rica, establishing the Monteverde Cloud Forest Preserve in 1972. Although this action seemed to secure the toad’s future, it is now apparent that setting aside habitat was not enough to save this beautiful creature. The toad’s demise in the late 1980s was a harbinger of further species extinction in Costa Rica. Since that time, another twenty of the fifty species of frogs and toads known to once inhabit a 30 square kilometer area near Monteverde have disappeared.
The unexplained, relatively sudden disappearance of amphibians in Costa Rica is not a unique story. Populations of frogs, toads, and salamanders have declined or disappeared the world over. Scientists hypothesize that the more subtle effects of human activities on the world’s ecosystems, such as the build-up of pollutants, the decrease in atmospheric ozone, and changing weather patterns due to global warming, are beginning to take their toll. Perhaps amphibians - whose permeable skin makes them sensitive to environmental changes - are the “canary in the coal mine,” giving us early notification of the deterioration of our environment. If amphibians are the biological harbingers of environmental problems, humans would be wise to heed their warning.
Q. 
It can be inferred from the passage that
  • a)
    only thirty species of frogs and toads remain in Costa Rica
  • b)
    humans do not have permeable skin
  • c)
    the build-up of pollutants in the atmosphere causes a decrease in atmospheric ozone
  • d)
    humans do not usually take signals of environmental deterioration seriously
  • e)
    Costa Rica suffers from more serious environmental problems than many other countries
Correct answer is option 'B'. Can you explain this answer?

Inference from the Passage
The correct answer is not option 'B', but let's explore the reasoning behind the inference options provided.
Understanding the Passage
The passage discusses the extinction of the golden toad in Costa Rica and the broader implications of amphibian population declines globally. It highlights human impacts on ecosystems, such as pollution and climate change, and emphasizes that amphibians, due to their sensitive skin, are particularly vulnerable to environmental changes.
Analysis of Options
- a) Only thirty species of frogs and toads remain in Costa Rica
- The passage states that twenty of the fifty species have disappeared, leaving thirty remaining, but it does not confirm that only thirty remain.
- b) Humans do not have permeable skin
- While the passage indicates that amphibians have permeable skin making them sensitive to changes, it does not discuss human skin. Therefore, this inference is not directly supported.
- c) The build-up of pollutants in the atmosphere causes a decrease in atmospheric ozone
- The passage mentions the build-up of pollutants and decreasing ozone, but it does not establish a direct cause-and-effect relationship.
- d) Humans do not usually take signals of environmental deterioration seriously
- The passage implies that humans may overlook the warning signs indicated by amphibians, making this inference plausible.
- e) Costa Rica suffers from more serious environmental problems than many other countries
- The passage does not make a comparative statement about Costa Rica's environmental issues relative to other countries.
Conclusion
Among these options, the most supported inference is option 'd', suggesting that humans may not take environmental warnings seriously. The passage serves as a call to recognize amphibians as indicators of broader ecological issues.

The ―paradox of tolerance‖ admonishes us that tolerance of the intolerant leads to intolerance. The Universal Declaration of Human Rights and the constitutions and laws of Western European democracies that adhere to the principle of freedom of speech all heed the warning of this conundrum and do not afford legal protection to extremist speech.  
While in Western European democracies, the speech of nondemocratic extremists has been successfully outlawed, in the United States the first amendment right to freedom of speech has been interpreted to encompass radical oration. The traditional justifications of this American stance originate in the belief that speech is entitled to greater tolerance than other kinds of activity. They are based on the belief that speech itself is valuable, and thus ascribe positive value to a very broad range of speech.
 According to the classical model, freedom of speech serves an indispensable function in the process of democratic self-government. From this perspective, the free speech principle need only protect political speech, comprised of all the facts, theories, and opinions relating to any issue on which the citizens must vote. Proponents of this view insist that even extremist views cannot be concealed from voting citizens, if these views bear on any public issue before them.
Protection of free speech serves the collective self-interests of a selfgoverning society made up of all rational, equal, and fully participating citizens who take their civic duties seriously. The fortress model is built on a foundation of pessimism, individualism, relativism, and self-doubt. At its deepest level, the fortress model values freedom of speech as a necessary precondition to the discovery and preservation of truth, but even at this level the function of speech remains primarily negative.  
From this perspective, the government and a majority of the people pose a great danger of intolerance. In spite of the high probability that their beliefs will eventually prove to be false, it is argued, people nonetheless tend to feel certain about them and, consequently, feel justified in requiring others to conform. Thus, the fortress model‘s prescription for combating the tendency to censor nonconforming views is to overprotect speech by providing a broad ―buffer zone‖ that encompasses extremist speech because its protection substantially diminishes the probability that inherently valuable speech will be suppressed.  
Directions: Read the above paragraph and answer the following
Q. The fortress model is ―built on a foundation of pessimism, individualism, relativism, and self-doubt.‖ Based on information in the passage, each of the following statements is a view held by those who believe in the fortress model of free speech EXCEPT:  
  • a)
    extremist political speech should be prohibited because it threatens democratic government.  
  • b)
    freedom of political speech is necessary in order to protect democratic government.  
  • c)
    a ban on extremist political speech raises the probability that more important political speech will also be banned.  
  • d)
    the government is unlikely to permit political speech that it finds objectionable unless the law prevents it from curbing political speech.  
  • e)
    the government should ensure that extremist speech, as long as it is political, is protected 
Correct answer is option 'A'. Can you explain this answer?

Arshiya Roy answered
Mapping the Passage
¶1 introduces the ―"paradox of intolerance."
¶2 gives examples of countries that have taken the paradox into consideration by banning extreme speech, and argues that the United States tolerates even extremist speech.
¶3 introduces a justification for the defense of extremist speech: the classical model.
¶s4 and 5 introduce another justification: the fortress model.
Review the fortress model before eliminating answer choices that match with what a proponent would believe. The fortress model argues that extremist speech should be protected because more harm is done in banning it than in allowing the speech itself. While three choices reasonably follow from this, (A) contradicts the main point of the model in general: free speech shouldn’t be banned.

Of course, in his attempts at field investigation, the historian is at the disadvantage that the countryside has changed in many respects since the period which he is studying. He is not permitted to use H.G. Wells‘s time machine, to enable him to see it as it actually was. Inevitably he is concerned in the main, if not exclusively, with literary and other materials, which have survived from that stretch of the past which interests him.
 Old maps may be plans of cities, charts of sea coasts and estuaries, cartularies of landed estates, or topographic delineations of land areas. These clearly engage the interest of historians and geographers alike, and they call for a combination of the methods and viewpoints of each. Maps can be conceived of and considered in several quite different ways, being properly regarded, and so assessed, as works of art—at best as objects of colour, skill, form, and beauty. They may alternatively be regarded purely for their cartographic aesthetic.  
The main queries which then arise are the following: how is it that the map-maker has carried out his task and with skill of what echelon and with what degree of success has he done so? Such an inquiry falls to the specialist field of historical cartography. An antiquarian map may also be approached in a means akin to that of the student who conceives it as a font contemporaneous with the time of its production. Thus, the historical cartographer may seek to bring grist to his mill and to consider the map‘s reliability as a satisfactory source of empirical evidence. By such means also the regional historian, in his search for essentials about such past matters as the availability of roads, the extent of enclosed farmland, or the number and location of mines and quarries, is no less an interested party.
 The value of old maps as documents useful for historicity depends necessarily on to what degree they depict and on how accurately. For virtually all periods of pre-modern history some maps have survived to serve as historiography, depicting, however imperfectly, certain features of past geography. The work of Claudius Ptolemy—who lived in the 2nd century A.D.—for centuries provided the basis for maps of the known world and its major regions. Although many were drawn on the scientific basis which he provided, they nevertheless embodied many errors—of location, distance, and the shape of areas of land and sea.  
The medieval portolan charts of the Mediterranean Sea and the later charts which provided sailing directions, produced in Holland, were accurate enough to be useful in practical navigation. Plans of important cities of Europe, so well-drawn as to yield evidence of their earlier form and extent, are notably offered in Braun and Hogenberg‘s Civitates Orbis Terrarum, published at Cologne and, in England, in John Speed‘s plans of cities. Similarly, John Ogilby‘s Britannia, Volume the First, appearing in 1675, gives detailed information of England's road system as it existed nearly three centuries ago. However, few of the early maps approach modern standards, which require accurate representation of distances and of heights above mean sea-level and the use of carefully distinguished symbols. This is because it was not until the 18th century that cartography, as an exact science, was born.  
Directions: Read the above paragraph and answer the following:
Q.With which of the following statements would the author be most likely to agree?  
  • a)
    Old maps provide important information about the past, even if they are somewhat misleading.  
  • b)
    Modern maps, in general, are more accurate than maps produced in the 18th century.  
  • c)
    The maps in Braun and Hogenberg‘s book have no historical value because of their errors.  
  • d)
    Claudius Ptolemy‘s maps were the most accurate ever made prior to the birth of modern cartography.                  
  • e)
    The field of cartography is on a downward spiral
Correct answer is option 'A'. Can you explain this answer?

Sakshi Das answered
Explanation:

Importance of Old Maps:
Old maps provide important information about the past, even if they are somewhat misleading. They serve as valuable historical documents that offer insights into the geography, topography, and features of earlier times.

Limited Accuracy:
While old maps may contain errors or inaccuracies, they still hold significance in understanding past landscapes and civilizations. Despite their limitations, they provide a glimpse into how people perceived and navigated the world in earlier centuries.

Historical Value:
Maps like those found in Braun and Hogenberg's Civitates Orbis Terrarum or John Speed's plans of cities may contain errors, but they are still valuable for studying the historical development of urban areas and landscapes.

Evolution of Cartography:
The author acknowledges that modern maps are more accurate than those produced in earlier centuries. However, the historical value of old maps lies in their ability to shed light on the past, even if they are not as precise as contemporary cartographic representations.

For millennia, the Nile River flooded nearly every year as a natural consequence of heavy summer rains on the Ethiopian Plateau; in the last century, as the population in the region exploded, the cycle of flooding interspersed with periodic drought caused widespread suffering for the local population.  In the mid-1950s, the Egyptian government concluded that a significant dam was necessary to enable the country’s economic development to be on a par with that of Western nations.  The Aswan Dam would prevent the annual flooding, generate hydroelectric power and supply a steady source of water for residents and agricultural activities, though it would also have other, less positive effects.
By the 1970s, most Egyptian villages had electric power, and the dam provided approximately half of Egypt’s entire output of electricity.  The benefits were counteracted, however, by consequences which were sometimes slow to appear but ruinous in their long-term effects.  Dams prevent silt from flowing through to downstream lands.
The silt is essential for renewing the minerals and nutrients that make the land fertile; before the dam, the Nile floodplain was famously productive.  Farmers have had to substitute artificial fertilizers, reducing profits and causing pervasive chemical pollution with deleterious effects for the human, animal and plant populations living near or in the river.  It is difficult to draw definite conclusions about a project with such substantial and varied results, but it would be untenable to assert that the Egyptian government should never have built the Aswan Dam.
Q.
Based upon the content of the passage, the author would most likely agree with which of the following propositions?
  • a)
    If a plan achieves its stated goals, it should still be carried out, even in the face of unintended negative results.
  • b)
    Planners of highly complex projects should expect some unintended negative consequences, even if they cannot foresee what those consequences will be.
  • c)
    Although a major undertaking may have unpredictable results, those results are not necessarily grounds for condemning the entire endeavor.
  • d)
    Any potential positive and negative effects should be weighed before starting a project of considerable magnitude or complexity.
  • e)
    It is necessary to determine the net impact of all outcomes, good and bad, before deciding whether to denounce the overall project.
Correct answer is option 'C'. Can you explain this answer?

The first paragraph of the passage introduces the reasons that the Egyptian government undertook to build the Aswan Dam and also lists the main benefits of the completed dam. The second paragraph begins with an example of a positive result, but then offers a significant and unexpected negative consequence.  It ends by stating that "it is difficult to draw... conclusions" when there are strong positive and negative effects, "but it would be untenable" to say that the dam shouldn't have been built.  The first half of this last sentence indicates the author's acknowledgment that this is a complex topic without any one right opinion or answer.  The second half, though, states that the author disagrees with those who believe the dam should not have been built. 
(A) The answer is too extreme; the author discusses only one example in the passage and does not make any sweeping conclusions.  He does not imply that anything that achieves its goals should be carried out.
(B) Although this may be a reasonable stance in the real world, it is out of scope.  The author does not discuss what planners should or should not expect anywhere in the passage. 
(C) CORRECT.  The passage essentially states that, despite mixed consequences, we cannot defend the position that the dam should not have been built, as the last sentence indicates that "it would be untenable to assert that the Egyptian government should never have built the Aswan Dam."  This mirrors the idea that "unpredictable" or mixed results do not necessarily lead to "condemning the entire endeavor."
(D) Although this may be a reasonable stance in the real world, it is out of scope.  Nowhere in the passage does the author discuss what actions should be taken before starting sizable projects.
(E) Although this may be a reasonable stance in the real world, it is out of scope.  The author does not discuss what criteria to use in order to decide whether to denounce a project.
In fact, the author states that it is "difficult to draw definite conclusions" even though the positive and negative outcomes are known in this circumstance. 

Measuring more than five feet tall and ten feet long, the Javan rhinoceros is often called the rarest large mammal on earth. None exist in zoos. Like the Indian rhino, the Javan has only one horn; African and Sumatran rhinos have two. While the Javan rhino habitat once extended across southern Asia, now there are fewer than one hundred of the animals in Indonesia and under a dozen in Vietnam. Very little is known about Javan rhinos because they lead secretive and solitary lives in remote jungles.
Until recently, scientists debated whether females even have horns, and most scientific work has had to rely on DNA garnered from dung.
The near extinction of the Javan rhino is the direct result of human actions. For centuries, farmers, who favored the same habitat, viewed them as crop eating pests and shot them on sight. During the colonial period, hunters slaughtered thousands. Now, human efforts to save them may well prove futile. The Vietnamese herd is probably doomed, as too few remain to maintain the necessary genetic variation. Rhinos from Java cannot supplement the Vietnamese numbers because in the millions of years since Indonesia separated from the mainland, the two groups have evolved into separate sub-species. In Indonesia, the rhinos are protected on the Ujung Kulon peninsula, which is unsettled by humans, and still have sufficient genetic diversity to have a chance at survival.
Ironically, however, the lack of human disturbance allows mature forests to replace the shrubby vegetation the animals prefer. Thus, human benevolence may prove little better for these rhinos than past human maltreatment.
Q.
According to the passage, which of the following best explains why the number of Javan rhinos in Vietnam cannot be increased by additions from those in the Ujung Kulon peninsula?
  • a)
    The Indonesian Javan rhinos constitute a separate sub-species.
  • b)
    The Javan rhinos cannot swim to Vietnam and have no land route available.
  • c)
    Neither Vietnam nor Indonesia has the funds for such a project.
  • d)
    Javan rhinos in the Ujung Kulon peninsula are almost impossible to capture.
  • e)
    Terrorist activity in Indonesia has made such a project too dangerous to attempt.
Correct answer is option 'A'. Can you explain this answer?

Prateek Gupta answered
The passage mentions the Ujung Kulon peninsula in the second paragraph. Immediately before that, the passage states that Indonesian Javan rhinos cannot supplement those in because they have evolved into separate sub-species. The correct answer choice will rely upon this fact.
(A) CORRECT. This choice rephrases the information in the second paragraph which states that the Indonesian Javan rhinos have evolved into a separate subspecies.
(B) The passage does not mention the aquatic abilities of the Javan rhino nor can it be assumed that they would have to get there without human assistance.
(C) The passage does not discuss the funds available in either country; therefore this choice is incorrect.
(D) The passage does not mention the difficulty involved in capturing a rhino; therefore this choice is incorrect.
(E) The passage does not mention terrorist activity at all; therefore, this choice is incorrect.  

Film scholars agree that Hollywood portrayals of America at war follow a cyclical pattern. During and immediately after a conflict, important films trumpet glory and sacrifice. Ten to fifteen years later, questioning and sometimes pacifistic movies about the conflict dominate. In the late 1960’s, “the raging bulls” of Hollywood—the young trendsetters rising to prominence—proclaimed this pattern obsolete. However, the passage of time has demonstrated this cultural pattern to be more resilient than it seemed in those days of social change.
Throughout the majority of the last century, evidence of the cyclical portrayal of war in film abounds. After America declared war against Germany during World War I, the still infant film industry glorified the fight against “the Hun.” By the early 1930’s, major releases had changed their tone; for example, All Quiet on the Western Front put forth an anti-war message by displaying the horrors of combat. After World War II began, the industry shifted gears. Suddenly, important pictures again portrayed glories and courage without the questioning or despair. For example, Guadalcanal Diary, produced during the war, showed “the ultimate sacrifice” as a noble and undoubted good. Once again, though, by 1957, films such as The Bridge on the River Kwai won awards for depicting the moral confusion of war.
Those who later declared this pattern dead based their conviction on their hearts rather than their minds. During the Vietnam War, the only major film about that conflict was The Green Berets, starring John Wayne and far closer in tone to Guadalcanal Diary than to The Bridge on the River Kwai. Similarly, years went by before more complex visions of war, such as Apocalypse Now, and then Platoon, emerged.
While today’s film industry is more diverse and its audience more culturally fragmented, this cycle largely continues. Jarhead, a layered depiction of the first gulf war, premiered more than ten years after that conflict. Further evidence of this pattern can be seen in the release of Apocalypse Now Redux, which contained additional footage that the producers originally thought would repel audiences. Thus, the famous aphorism “The more things change, the more they stay the same” certainly applies to this aspect of the film industry.
Q.
What is the main point made by the author?
  • a)
    Hollywood has never fully supported America’s armed conflicts.
  • b)
    In the last century, the film industry has become more culturally diverse.
  • c)
    An established cultural pattern is more durable than was thought during a time of social upheaval.
  • d)
    The film industry has only supported American military efforts during the actual conflict.
  • e)
    Cyclical patterns determine the type of big budget films produced by Hollywood more than individuals do.
Correct answer is option 'C'. Can you explain this answer?

Mehul Nair answered
The passage clearly enunciates in the first paragraph that it plans to illustrate the cyclical pattern of the tone of Hollywood war movies. The second and third paragraphs trace the pattern's history through the last century, and then the passage ends by returning to the resilience of that cycle. Thus, the main point of the passage must reference establishing and describing the recurring pattern over time. 
(A) This choice does not address the cyclical pattern; instead, it suggests a point not made in the passage.
(B) This is a minor detail mentioned in the last paragraph, not the main point. Furthermore, this choice ignores the issue of a cyclical pattern.
(C) CORRECT. This choice reiterates the theme that a pattern is durable, despite the doubts of some during "those days of social change" -- i.e., the late 1960's. This choice exactly mirrors the structure of the passage, which makes this point, provides historical evidence, and then reiterates that the pattern continues to endure.
(D) Besides its extreme quality, this choice is incorrect because it distorts the meaning. The passage's assertion that a more nuanced view of America’s conflicts emerges in movies made years later cannot automatically be equated with a lack of support.
(E) This choice presents an irrelevant comparison. The passage is only concerned with the existence of this pattern and does not address the relative influence of various individuals. 

For years, U.S. employers have counted on a steady flow of labor from Mexico willing to accept low-skilled, low paying jobs. These workers, many of whom leave economically depressed villages in the Mexican interior, are often more than willing to work for wages well below both the U.S. minimum wage and the poverty line.
However, thanks to a dramatic demographic shift currently taking place in Mexico, the seemingly inexhaustible supply of workers migrating from Mexico to the United States might one day greatly diminish if not cease.
Predictions of such a drastic decrease in the number of Mexican immigrants, both legal and illegal, are driven by Mexico’s rapidly diminishing population growth. As a result of a decades-long family planning campaign, most Mexicans are having far fewer children than was the norm a generation ago. The campaign, organized around the slogan that “the small family lives better,” saw the Mexican government establish family-planning clinics and offer free contraception. For nearly three decades, the government’s message concerning population hasn’t wavered. In fact, the Mexican Senate recently voted to extend public school sex education programs to kindergarten.
The result of Mexico’s efforts to stem population growth is nothing short of stunning. In 1968, the average Mexican woman had just fewer than seven children; today, the figure is slightly more than two. For two primary reasons, Mexico’s new demographics could greatly impact the number of Mexicans seeking work in the U.S. First, smaller families by their nature limit the pool of potential migrants.
Second, the slowing of Mexico’s population growth has fostered hope that Mexico will develop a healthy middle class of people content to make their livelihoods in their home country.
Though the former of these factors is all but assured, the growth of a healthy middle class is far from a foregone conclusion. The critical challenge for Mexico is what it does with the next 20 years. Mexico must invest in education, job training, and infrastructure, as well as a social-security system to protect its aging population.  If Mexico is willing to step forward and meet this challenge, America may one day wake up to find that, like cheap gasoline, cheap Mexican labor has become a thing of the past.
Q.
With which of the following statements would the author of the passage MOST likely agree?
  • a)
    The United States will soon have to replace lost Mexican labor with labor provided by other immigrant groups.
  • b)
    It is difficult for a country with a large population to develop a healthy middle class.
  • c)
    Many Mexican immigrants who work in the United States believe that they are taken advantage of by American employers.
  • d)
    Most rapidly growing countries should institute a family planning campaign to limit population growth.
  • e)
    Mexico does not currently have the infrastructure to develop a healthy middle class.
Correct answer is option 'E'. Can you explain this answer?

Kalyan Nair answered
A keen understanding of the main idea of the passage, in combination with a well developed sketch of the passage to help you quickly access the pertinent information, will help you to accurately eliminate incorrect answer choices.
(A) In both the first and fourth paragraphs the author indicates that the supply of Mexican labor to the United States might diminish over time if certain conditions in Mexico continue to change. The author does not believe this to be a foregone conclusion. Moreover, the author never hints that this hypothetical labor vacuum will have to be filled by other immigrant groups.
(B) The author speaks only of Mexico and its challenges. It is beyond the scope of the passage to assume what the author might believe, in general, about countries with large populations. Additionally, the passage never quantifies the definition of a “large population.”  
(C) In the first paragraph, the author says Mexican immigrants are “more than willing to work for wages well below both the U.S. minimum wage and the poverty line.” Thus, the workers themselves seem not to feel taken advantage of.
(D) Similar to answer choice B, this answer choice asks about the author’s attitude toward countries other than Mexico. The passage, however, discusses only Mexico and its policies and challenges.
(E) CORRECT. In the final paragraph, the author indicates that "the growth of a healthy middle class is far from a foregone conclusion. The critical challenge for Mexico is what it does with the next 20 years.” Then, the author indicates the steps that Mexico must take. Thus, the author obviously believes that Mexico does not currently have the infrastructure to develop a healthy middle class. The key, for the author, is whether Mexico is willing and able to build this infrastructure.  

In public Greek life, a man had to make his way at every step through the immediate persuasion of the spoken word. Whether it be addressing an assembly, a law-court or a more restricted body, his oratory would be a public affair rather than under the purview of a quiet committee, without the support of circulated commentary, and with no backcloth of daily reportage to make his own or others‘ views familiar to his hearers. The oratory's immediate effect was all-important; it would be naive to expect that mere reasonableness or an inherently good case would equate to a satisfactory appeal. Therefore, it was early realized that persuasion was an art, up to a point teachable, and a variety of specific pedagogy was well established in the second half of the fifth century. When the sophists claimed to teach their pupils how to succeed in public life, rhetoric was a large part of what they meant, though, to do them justice, it was not the whole.
Skill naturally bred mistrust. If a man of good will had need of expression advanced of mere twaddle, to learn how to expound his contention effectively, the truculent or pugnacious could be taught to dress their case in well-seeming guise. It was a standing charge against the sophists that they 'made the worse appear the better cause,‘ and it was this immoral lesson which the hero of Aristophanes‘ Clouds went to learn from, of all people, Socrates. Again, the charge is often made in court that the opponent is an adroit orator and the jury must be circumspect so as not to let him delude them. From the frequency with which this crops up, it is patent that the accusation of cleverness might damage a man. In Greece, juries, of course, were familiar with the style, and would recognize the more evident artifices, but it was worth a litigant‘s while to get his speech written for him by an expert. Persuasive oratory was certainly one of the pressures that would be effective in an Athenian law-court.
A more insidious danger was the inevitable desire to display this art as an art. It is not easy to define the point at which a legitimate concern with style shades off into preoccupation with manner at the expense of matter, but it is easy to perceive that many Greek writers of the fourth and later centuries passed that danger point. The most influential was Isocrates, who polished for long years his pamphlets, written in the form of speeches, and taught to many pupils the smooth and easy periods he had perfected. Isocrates took to the written word in compensation for his inadequacy in live oratory; the tough and nervous tones of a Demosthenes were far removed from his, though they, too, were based on study and practice. The exaltation of virtuosity did palpable harm. The balance was always delicate, between style as a vehicle and style as an end in itself.
 We must not try to pinpoint a specific moment when it, once and for all, tipped over; but certainly, as time went on, virtuosity weighed heavier. While Greek freedom lasted, and it mattered what course of action a Greek city decided to take, rhetoric was a necessary preparation for public life, whatever its side effects. It had been a source of strength for Greek civilization that its problems, of all kinds, were thrashed out very much in public. The shallowness which the study of rhetoric might (not must) encourage was the corresponding weakness.  
Directions: Read the above paragraph and answer the following
Q.Implicit in the statement that the exaltation of virtuosity was not due mainly to Isocrates because public display was normal in a world that talked far more than it read is the assumption that:  
  • a)
    Isocrates was actually concerned as much with the content of his speeches as with their style.  
  • b)
    excessive concern with style is bound to arise in a world dominated by public display.  
  • c)
    the Greeks were guilty of exalting virtuosity in their public art and architecture as well.  
  • d)
    Isocrates was less influential than previous historians estimated.        
  • e)
    there should be no connection between communication style and public display of thoughts 
Correct answer is option 'B'. Can you explain this answer?

Dhruba Jain answered
Mapping the Passage:
¶1 introduces the importance of rhetoric in Greek life and the fact that it was taught.
¶2 explains why rhetorical skill was sometimes mistrusted, but still sought after.
¶3 states that as rhetoric (in writing and speech) became more of an art, its original purpose was corrupted.
¶4 states that the Greek system required rhetorical skills and therefore inherited rhetoric's drawbacks as well.
Make sure that you untangle tough questions, paraphrasing what‘s being asked, before trying to answer them. What paragraph is being discussed? ¶3, the argument that the art of rhetoric became too important. The question stem just says that this happened because the culture was concerned with public display. Assumptions bridge gaps in reasoning. Here, it would connect art and public display. Only (B) and (C) deal with both of these concepts. If (B) is true, we have a valid explanation for why art became so important in this particular culture. If it‘s not true, there‘s no reason why they should be connected, and the author‘s argument falls apart. (B) has to be a valid assumption

Direction For questions : Scientists have long recognized the incredible diversity within a species. But they thought it reflected evolutionary changes that unfolded imperceptibly, over millions of years. That divergence between populations within a species was enforced, according to Ernst Mayr, the great evolutionary biologist of the 1940s, when a population was separated from the rest of the species by a mountain range or a desert, preventing breeding across the divide over geologic scales of time. Without the separation, gene flow was relentless. But as the separation persisted, the isolated population grew apart and speciation occurred. In the mid-1960s, the biologist Paul Ehrlich – author of The Population Bomb (1968) – and his Stanford University colleague Peter Raven challenged Mayr’s ideas about speciation. They had studied checkerspot butterflies living in the Jasper Ridge Biological Preserve in California, and it soon became clear that they were not examining a single population. Through years of capturing, marking and then recapturing the butterflies, they were able to prove that within the population, spread over just 50 acres of suitable checkerspot habitat, there were three groups that rarely interacted despite their very close proximity. Among other ideas, Ehrlich and Raven argued in a now classic paper from 1969 that gene flow was not as predictable and ubiquitous as Mayr and his cohort maintained, and thus evolutionary divergence between neighbouring groups in a population was probably common. They also asserted that isolation and gene flow were less important to evolutionary divergence than natural selection (when factors such as mate choice, weather, disease or predation cause better-adapted individuals to survive and pass on their successful genetic traits). For example, Ehrlich and Raven suggested that, without the force of natural selection, an isolated population would remain unchanged and that, in other scenarios, natural selection could be strong enough to overpower gene flow...
Q. The author discusses Mayr, Ehrlich and Raven to demonstrate that:
  • a)
    evolution is a sensitive and controversial topic.
  • b)
    Ehrlich and Raven’s ideas about evolutionary divergence are widely accepted by scientists.
  • c)
    the causes of speciation are debated by scientists.
  • d)
    checkerspot butterflies offer the best example of Ehrlich and Raven’s ideas about speciation.
Correct answer is option 'A'. Can you explain this answer?

Aditya Kumar answered
Option 1 is the correct answer as the author provides examples of the US and Russia to tell that these countries have taken up the use of typewriter. The author also supports their using of typewriters by providing us with positive aspects of the typewriters. Option 2 is beyond the scope of the passage. Option 3 is incorrect since clearly it is not the main aim of the passage. There is no hint to make that claim. Option 4 is incorrect since this difference has nowhere been made.

Direction For question : As the Heart of Darkness makes its way into the savage shadows of the African continent, Joseph Conrad exposes a psycho-geography of the collective unconscious in the entangling symbolic realities of the serpentine Congo. Conrad’s novella descends into the unknowable darkness at the heart of Africa, taking its narrator, Marlow, on an underworld journey of individuation, a modern odyssey toward the center of the Self and the center of the Earth. Ego dissolves into soul as, in the interior; Marlow encounters his double in the powerful image of ivory-obsessed Kurtz, the dark shadow of European imperialism. The dark meditation is graced by personifications of anima in Kurtz’ black goddess, the savagely magnificent consort of the underworld, and in his porcelain -skinned Persephone, innocent intended of the upper world. “Africa,” wrote Graham Greene, “will always be the Africa in the Victorian atlas, the blank unexplored continent in the shape of the human heart.” The African heart described by Greene “acquired a new layer of meaning when Conrad portrayed the Congo under King Leopold as the Heart of Darkness, a place where barbarism triumphs over humanity, nature over technology, biology over culture, id over super ego.” The unknown and uncharted topography of the African continent first beckoned Conrad’s narrator, Marlow, into its depths in his boyhood: “Now, when I was a little chap I had a passion for maps. I would look for hours at South America, or Africa, or Australia, and lose myself in all the glories of exploration”. When Marlow was grown and Africa was no longer a blank space on the map, but rather “a place of darkness,” there was still one river there that drew him especially, “a mighty big river, that you could see on the map, resembling an immense snake uncoiled, with its head in the sea, its body at rest curving afar over a vast country, and its tail lost in the depths of the land”. This same deep place, the Congo, that had seduced Conrad’s ivory hunting Kurtz into the horrors of its savage embrace had, in 1890, lured Conrad himself into adventure that turned him from sailor to writer and severely affected his health for the rest of his life. As the voyage up the Congo pro ved fateful for the development of Conrad’s narrator, Marlow, it was equally fateful for Conrad’s individuation, as he reflects in his letters “Before the Congo I was just a mere animal.” Hillman, in “Notes on White Supremacy” reminds us that, like Conrad, both Freud and Jung were called to venture into the shadowed continent and vestiges of their journeys still colour our psychological language: The convention informing geographical discoveries and the expansion of white consciousness over Africa continue to inform psychic geography. The topological language used by Freud for “the unconscious” as a place below, different, timeless, primordial, libidinal and separated from the consciousness recapitulates what white reporters centuries earlier said about West Africa. From Conrad’s Heart of Darkness to van der Post’s Venture to the Interior, Africa and the unconscious allegorize the other place.... “Just don’t stay in the topical colonies too long; you must reign at home,” writes Freud in 1911 to Jung, who himself made the African journey fourteen years later, describing the vast lands and dark peoples he encountered in language he applies as well to the immemorial unconscious psyche.... Part of psychology’s myth is that the unconscious was “discovered” as its contents are “explored”. Thus Africa has become a topology of the mind — its location, its shape, its cultures, its textures, its rhythms, its foliage, its hues, its wildness — all calling forth something lost in the psychology of the white European. It is with an understanding of our destiny to explore that symbolic lost continent within ourselves that we can begin to appreciate the prescience of Jungian psychology in Conrad’s Heart of Darkness.
Q. Which of the following is most likely to be the profession of the author?
  • a)
    A movie critic
  • b)
    A newspaper analyst
  • c)
    A college Professor
  • d)
    An English literature scholar
Correct answer is option 'D'. Can you explain this answer?

Shalini Patel answered
The author can be a professor but the subject is missing. Hence, the most likely answer is 4 where the author’s background in English literature is clearly visible.

For millennia, the Nile River flooded nearly every year as a natural consequence of heavy summer rains on the Ethiopian Plateau; in the last century, as the population in the region exploded, the cycle of flooding interspersed with periodic drought caused widespread suffering for the local population.  In the mid-1950s, the Egyptian government concluded that a significant dam was necessary to enable the country’s economic development to be on a par with that of Western nations.  The Aswan Dam would prevent the annual flooding, generate hydroelectric power and supply a steady source of water for residents and agricultural activities, though it would also have other, less positive effects.
By the 1970s, most Egyptian villages had electric power, and the dam provided approximately half of Egypt’s entire output of electricity.  The benefits were counteracted, however, by consequences which were sometimes slow to appear but ruinous in their long-term effects.  Dams prevent silt from flowing through to downstream lands.
The silt is essential for renewing the minerals and nutrients that make the land fertile; before the dam, the Nile floodplain was famously productive.  Farmers have had to substitute artificial fertilizers, reducing profits and causing pervasive chemical pollution with deleterious effects for the human, animal and plant populations living near or in the river.  It is difficult to draw definite conclusions about a project with such substantial and varied results, but it would be untenable to assert that the Egyptian government should never have built the Aswan Dam.
Q.
All of the following are mentioned as benefits of the Aswan Dam EXCEPT
  • a)
    a constant source of fresh water for consumers and industry
  • b)
    electrical power for a sizable portion of the population
  • c)
    the facilitation of economic development possibilities similar to those in Western nations
  • d)
    the elimination of annual flood cycles in the region
  • e)
    the protection of animal and plant life in and near the river
Correct answer is option 'E'. Can you explain this answer?

Aditya Sharma answered
Specific detail EXCEPT questions require us to search the passage for four details which are mentioned in the passage; the fifth is not mentioned and is, therefore, the right answer.  One helpful technique is to label each answer choice with a T (for "true") if you find it in the passage and an F (for "false") if you cannot.
(A) True.  The last sentence of the first paragraph says the dam will "supply a steady source of water for residents and agricultural activities."
(B) True.  The first sentence of the second paragraph says "the dam provided approximately half of Egypt's entire electricity output.
(C) True.  The second sentence of the first paragraph says that the dam would "enable the country's economic development to be on a par with that of Western nations," and it is clear from the passage that the actual completion of the dam served to move Egypt toward this goal
(D) True.  The last sentence of the first paragraph says the dam "would prevent the annual flooding" and the last sentence of the second paragraph confirms that the dam succeeded in this goal.
(E) CORRECT.  False.  In fact, the opposite is true; the dam led farmers to use fertilizers which caused pollution and harmed the animal and plant life in the area. 

Film scholars agree that Hollywood portrayals of America at war follow a cyclical pattern. During and immediately after a conflict, important films trumpet glory and sacrifice. Ten to fifteen years later, questioning and sometimes pacifistic movies about the conflict dominate. In the late 1960’s, “the raging bulls” of Hollywood—the young trendsetters rising to prominence—proclaimed this pattern obsolete. However, the passage of time has demonstrated this cultural pattern to be more resilient than it seemed in those days of social change.
Throughout the majority of the last century, evidence of the cyclical portrayal of war in film abounds. After America declared war against Germany during World War I, the still infant film industry glorified the fight against “the Hun.” By the early 1930’s, major releases had changed their tone; for example, All Quiet on the Western Front put forth an anti-war message by displaying the horrors of combat. After World War II began, the industry shifted gears. Suddenly, important pictures again portrayed glories and courage without the questioning or despair. For example, Guadalcanal Diary, produced during the war, showed “the ultimate sacrifice” as a noble and undoubted good. Once again, though, by 1957, films such as The Bridge on the River Kwai won awards for depicting the moral confusion of war.
Those who later declared this pattern dead based their conviction on their hearts rather than their minds. During the Vietnam War, the only major film about that conflict was The Green Berets, starring John Wayne and far closer in tone to Guadalcanal Diary than to The Bridge on the River Kwai. Similarly, years went by before more complex visions of war, such as Apocalypse Now, and then Platoon, emerged.
While today’s film industry is more diverse and its audience more culturally fragmented, this cycle largely continues. Jarhead, a layered depiction of the first gulf war, premiered more than ten years after that conflict. Further evidence of this pattern can be seen in the release of Apocalypse Now Redux, which contained additional footage that the producers originally thought would repel audiences. Thus, the famous aphorism “The more things change, the more they stay the same” certainly applies to this aspect of the film industry.
Q.
Which one of the following does the author believe is true about The Bridge on the River Kwai?
  • a)
    it deserved the awards that it won.
  • b)
    It is a more intelligent and well-crafted movie than The Green Berets.
  • c)
    It was the first movie to portray the moral confusion of war.
  • d)
    Its portrayal of war is more ambivalent than that in Guadalcanal Diary.
  • e)
    It was more financially successful than any war movie that came before it.
Correct answer is option 'D'. Can you explain this answer?

In the second paragraph, The Bridge on the River Kwai is mentioned as a post-WWII example of the continuing pattern of war movies becoming more ambivalent years after the conflict. The phrases “once again” and “for example” highlight this. In the third paragraph, the movie is again mentioned as an example of a more complex view of war. Thus, the correct answer must follow from these points.
(A) This is incorrect; the author does not discuss the quality of the movie or the merit of any awards. 
(B) The passage does not compare the intelligence or crafting of these, or any, films; it merely discusses their tone and approach to the depiction of war.  A movie could present a glorified depiction of war and also be very intelligent and well-crafted.
(C) This is incorrect. The passage indicates that there were movies with ambivalent perspectives produced after the first war, citing All Quiet on the Western Front as an example. Furthermore, the phrase “once again” indicates that it was not the first to have such a tone. 
(D) CORRECT. The third paragraph states that Guadalcanal Diary was much closer in tone to The Green Berets and contrasts those two to The Bridge on the River Kwai. This contrast is in terms of the greater ambivalence and moral confusion shown in The Bridge on the River Kwai.
(E) The passage does not mention the financial success, or lack thereof, of any of the movies it discusses. 

Film scholars agree that Hollywood portrayals of America at war follow a cyclical pattern. During and immediately after a conflict, important films trumpet glory and sacrifice. Ten to fifteen years later, questioning and sometimes pacifistic movies about the conflict dominate. In the late 1960’s, “the raging bulls” of Hollywood—the young trendsetters rising to prominence—proclaimed this pattern obsolete. However, the passage of time has demonstrated this cultural pattern to be more resilient than it seemed in those days of social change.
Throughout the majority of the last century, evidence of the cyclical portrayal of war in film abounds. After America declared war against Germany during World War I, the still infant film industry glorified the fight against “the Hun.” By the early 1930’s, major releases had changed their tone; for example, All Quiet on the Western Front put forth an anti-war message by displaying the horrors of combat. After World War II began, the industry shifted gears. Suddenly, important pictures again portrayed glories and courage without the questioning or despair. For example, Guadalcanal Diary, produced during the war, showed “the ultimate sacrifice” as a noble and undoubted good. Once again, though, by 1957, films such as The Bridge on the River Kwai won awards for depicting the moral confusion of war.
Those who later declared this pattern dead based their conviction on their hearts rather than their minds. During the Vietnam War, the only major film about that conflict was The Green Berets, starring John Wayne and far closer in tone to Guadalcanal Diary than to The Bridge on the River Kwai. Similarly, years went by before more complex visions of war, such as Apocalypse Now, and then Platoon, emerged.
While today’s film industry is more diverse and its audience more culturally fragmented, this cycle largely continues. Jarhead, a layered depiction of the first gulf war, premiered more than ten years after that conflict. Further evidence of this pattern can be seen in the release of Apocalypse Now Redux, which contained additional footage that the producers originally thought would repel audiences. Thus, the famous aphorism “The more things change, the more they stay the same” certainly applies to this aspect of the film industry.
Q.
In the second paragraph, the author implies that “the Hun” refers to which of the following?
  • a)
    The Huns
  • b)
    The Hungarians
  • c)
    The Austro-Hungarians
  • d)
    The Germans
  • e)
    The Russians
Correct answer is option 'D'. Can you explain this answer?

Saumya Shah answered
he second sentence of the second paragraph reads “After America declared war against Germany during World War I, the still infant film industry glorified the fight against “the Hun.” Thus, it must follow that the Germans are referred to by that name.  GMAT inferences generally follow directly from the text of the passage.   
(A) There is no direct evidence in the passage that indicates this meaning for "the Hun." Furthermore, the use of quotation marks indicates that the reference is not literal.
(B) There is no direct evidence in the passage that indicates this meaning for "the Hun."  
(C) There is no direct evidence in the passage that indicates this meaning for "the Hun."  The fact that the Hungarians were then part of Austro-Hungary is outside knowledge which cannot be deduced from the passage.
(D) CORRECT. Since the passage states that America declared war on Germany, it must follow that the Germans were “the Hun” against whom America was fighting.
(E) There is no direct evidence in the passage that indicates this meaning for "the Hun." GMAT inferences must follow from the text of the passage. 

In the past decade, rapid technological progress and a greater demand for high-quality digital imaging have led to dramatic advances in video display technology. The dominant technology currently used in most consumer product displays is the active matrix liquid crystal diode display (LCD).
LCDs apply thin-film transistors (TFTs) of amorphous or polycrystalline silicon sandwiched between two glass plates. The TFTs supply voltage to liquid-crystal-filled cells, or pixels, between the sheets of glass. When hit with an electric charge, the liquid crystals untwist to an exact degree to filter white light generated by a lamp. This filtered light shines directly on the viewing screen or, in the case of projection televisions, is projected through a small chip that acts as a lens. LCDs that are capable of producing color images, such as in televisions and computers, reproduce colors through a process of subtraction, blocking out particular color wavelengths from the spectrum of white light until only the desired color remains. It is the variation of the intensity of light permitted to pass through the matrix of liquid crystals that enables LCD displays to present images full of gradations of different colors.
The nature and functioning of LCD displays present many advantages relative to other display technologies. The amount of power required to untwist the crystals to display images, even dark ones, is much lower than that required for analogous processes using other technologies, such as plasma. The dense array of crystals displays images from computer or other video graphics sources extremely well, with full color detail, no flicker, and no screen burnin. Moreover, the number of pixels per square inch on an LCD display is typically higher than that for other display technologies, so LCD monitors are particularly good at displaying large amounts of data with exceptional clarity and precision. As a result, LCD TVs are considered the best display platform for video games, high definition television, movie special effects, and other graphicsintensive uses.
Q.
The process through which an LCD monitor displays different colors is most closely analogous to
  • a)
    An hourglass partially blocked such that a limited stream of grains of sand fall into the lower portion
  • b)
    A series of filters that separate all of the components of a mixture according to size
  • c)
    A recording st udi o so undproo fed s o t ha t any performances within are muted to those outside
  • d)
    A pi ece of construction paper with outline s of characters cut out such that a lamp in front of the paper casts shadows in the shapes of the characters
  • e)
    An air vent that emits warmer air outside of a building while an air conditioning system cools the interior of the building
Correct answer is option 'D'. Can you explain this answer?

Rhea Gupta answered
This question asks which is most analogous to the process through which an LCD display presents different colors. The fifth and sixth sentences of the second paragraph read, “LCDs that are capable of producing color images, such as in televisions and computers, reproduce colors through a process of subtraction, blocking out particular color wavelengths from the spectrum of white light until only the desired color remains. It is the variation of the intensity of light permitted to pass through the matrix of liquid crystals that enables LCD displays to present images full of gradations of different colors.” This process of subtraction consists of taking a large number of elements, in this case wavelengths of light, and carefully blocking out certain of them while allowing only certain others to get through. We should look for an answer that mirrors this precise process of subtraction.
(A) This answer choice describes a process that does not result in only certain elements passing through. Each grain of sand is equally capable of falling through the hourglass; in fact, all of it will eventually pass through, although it will happen at a slower rate than it might because it the opening is partially blocked.
(B) A series of filters that separate out all of the elements of a mixture is not analogous to filtering out only certain elements and letting others "pass through" to together create the end result.
(C) A soundproofed recording studio blcoks all sound, not only certain elements of it. Hence, this is very different from the process described regarding an LCD display. 
(D) CORRECT. A piece of construction paper with outlines of characters cut out would allow certain spaces of light to pass while blocking others, resulting in a refined mixture that has a certain meaning (e.g. the word, “Apple”). This is closely analogous to the process of subtraction through which LCD displays different colors. (E) An air vent that expels warm air does not carefully select remaining elements, and all of the elements that pass through are indistinguishable from each other. 

For years, U.S. employers have counted on a steady flow of labor from Mexico willing to accept low-skilled, low paying jobs. These workers, many of whom leave economically depressed villages in the Mexican interior, are often more than willing to work for wages well below both the U.S. minimum wage and the poverty line.
However, thanks to a dramatic demographic shift currently taking place in Mexico, the seemingly inexhaustible supply of workers migrating from Mexico to the United States might one day greatly diminish if not cease.
Predictions of such a drastic decrease in the number of Mexican immigrants, both legal and illegal, are driven by Mexico’s rapidly diminishing population growth. As a result of a decades-long family planning campaign, most Mexicans are having far fewer children than was the norm a generation ago. The campaign, organized around the slogan that “the small family lives better,” saw the Mexican government establish family-planning clinics and offer free contraception. For nearly three decades, the government’s message concerning population hasn’t wavered. In fact, the Mexican Senate recently voted to extend public school sex education programs to kindergarten.
The result of Mexico’s efforts to stem population growth is nothing short of stunning. In 1968, the average Mexican woman had just fewer than seven children; today, the figure is slightly more than two. For two primary reasons, Mexico’s new demographics could greatly impact the number of Mexicans seeking work in the U.S. First, smaller families by their nature limit the pool of potential migrants.
Second, the slowing of Mexico’s population growth has fostered hope that Mexico will develop a healthy middle class of people content to make their livelihoods in their home country.
Though the former of these factors is all but assured, the growth of a healthy middle class is far from a foregone conclusion. The critical challenge for Mexico is what it does with the next 20 years. Mexico must invest in education, job training, and infrastructure, as well as a social-security system to protect its aging population.  If Mexico is willing to step forward and meet this challenge, America may one day wake up to find that, like cheap gasoline, cheap Mexican labor has become a thing of the past.
Q.
Which of the following can be inferred about U.S. employers of Mexican immigrants?
  • a)
    Most of these employers pay Mexican immigrants less money than they pay American citizens.
  • b)
    Some of these employers violate wage laws.
  • c)
    Many of these employers work in the agricultural industry.
  • d)
    Whitout Mexican  immigrants, some of these employers would be forced to close their businesses.
  • e)
    The majority of these employers show no concern for the welfare of their workers.
Correct answer is option 'B'. Can you explain this answer?

Moumita Sen answered
The correct answer to an inference question must be directly supported by evidence from the text. The passage states that U.S. employers of Mexican immigrants often provide low-skilled, low-paying jobs to individuals who "are often more than willing to work for wages well below both the U.S. minimum wage and the poverty line."
(A) No information is provided concerning the amount of money paid to American citizens.  It is possible that there are many Americans who also work for wages well below both the U.S. minimum wage and the poverty line.    
(B) CORRECT. If some of these immigrant workers are accepting wages “well below the U.S. minimum wage,” their American employers must be violating wage laws (i.e. paying wages below what the U.S. minimum wage requires).  
(C) Nothing in the passage suggests the particular industry of these employers. This answer is outside the scope of the argument, and assumes knowledge from sources other than the passage.    
(D) The passage does not suggest that, without labor from Mexico, these employers will be forced to close. This answer is both too predictive and outside the scope of the argument.  
(E) The passage suggests nothing about how these employers either regard or treat their workers. Moreover, indication that these employers show “no concern” is too extreme to be inferred from the passage. 

Film scholars agree that Hollywood portrayals of America at war follow a cyclical pattern. During and immediately after a conflict, important films trumpet glory and sacrifice. Ten to fifteen years later, questioning and sometimes pacifistic movies about the conflict dominate. In the late 1960’s, “the raging bulls” of Hollywood—the young trendsetters rising to prominence—proclaimed this pattern obsolete. However, the passage of time has demonstrated this cultural pattern to be more resilient than it seemed in those days of social change.
Throughout the majority of the last century, evidence of the cyclical portrayal of war in film abounds. After America declared war against Germany during World War I, the still infant film industry glorified the fight against “the Hun.” By the early 1930’s, major releases had changed their tone; for example, All Quiet on the Western Front put forth an anti-war message by displaying the horrors of combat. After World War II began, the industry shifted gears. Suddenly, important pictures again portrayed glories and courage without the questioning or despair. For example, Guadalcanal Diary, produced during the war, showed “the ultimate sacrifice” as a noble and undoubted good. Once again, though, by 1957, films such as The Bridge on the River Kwai won awards for depicting the moral confusion of war.
Those who later declared this pattern dead based their conviction on their hearts rather than their minds. During the Vietnam War, the only major film about that conflict was The Green Berets, starring John Wayne and far closer in tone to Guadalcanal Diary than to The Bridge on the River Kwai. Similarly, years went by before more complex visions of war, such as Apocalypse Now, and then Platoon, emerged.
While today’s film industry is more diverse and its audience more culturally fragmented, this cycle largely continues. Jarhead, a layered depiction of the first gulf war, premiered more than ten years after that conflict. Further evidence of this pattern can be seen in the release of Apocalypse Now Redux, which contained additional footage that the producers originally thought would repel audiences. Thus, the famous aphorism “The more things change, the more they stay the same” certainly applies to this aspect of the film industry.
Q.
What is the function of the last paragraph of the passage?
  • a)
    It shows that, despite changes in the industry and audience, the pattern discussed still exists.
  • b)
    t points out that the film industry never changes.
  • c)
    It shows that changes in the film industry and its audience have made the pattern previously discussed obsolete.
  • d)
    It discusses how Jarhead and Apocalypse Now Redux are fundamentally different from all the war movies that preceded them.
  • e)
    It demonstrates that war movies have changed to reflect the more culturally diverse audience.
Correct answer is option 'A'. Can you explain this answer?

Ameya Yadav answered
The last paragraph begins by noting that the changes in the industry and audience have not changed the pattern discussed. The paragraph ends by stating that this aspect of the film industry, that is, the pattern discussed, remains intact. The correct answer should reflect this point.
(A) CORRECT. This choice correctly summarizes the point made in the first and last sentences of the last paragraph: that this pattern continues despite changes in the industry and audience. 
(B) This choice is too extreme. The passage merely says that one pattern of the film industry has not changed. The last paragraph acknowledges that changes in the industry have occurred. 
(C) This choice is incorrect as the last paragraph explicitly states that the pattern discussed largely continues. 
(D) In the last paragraph, Jarhead and Apocalypse Now Redux are mentioned as examples proving that the discussed pattern continues. Thus, they cannot be fundamentally different from other war movies.
(E) The point of the last paragraph is that the pattern of the last century continues. Thus, it cannot make the point that war movies have changed in response to changes in the audience. 

For millennia, the Nile River flooded nearly every year as a natural consequence of heavy summer rains on the Ethiopian Plateau; in the last century, as the population in the region exploded, the cycle of flooding interspersed with periodic drought caused widespread suffering for the local population.  In the mid-1950s, the Egyptian government concluded that a significant dam was necessary to enable the country’s economic development to be on a par with that of Western nations.  The Aswan Dam would prevent the annual flooding, generate hydroelectric power and supply a steady source of water for residents and agricultural activities, though it would also have other, less positive effects.
By the 1970s, most Egyptian villages had electric power, and the dam provided approximately half of Egypt’s entire output of electricity.  The benefits were counteracted, however, by consequences which were sometimes slow to appear but ruinous in their long-term effects.  Dams prevent silt from flowing through to downstream lands.
The silt is essential for renewing the minerals and nutrients that make the land fertile; before the dam, the Nile floodplain was famously productive.  Farmers have had to substitute artificial fertilizers, reducing profits and causing pervasive chemical pollution with deleterious effects for the human, animal and plant populations living near or in the river.  It is difficult to draw definite conclusions about a project with such substantial and varied results, but it would be untenable to assert that the Egyptian government should never have built the Aswan Dam.
Q.
TCrops cannot grow without silt.he passage implies which of the following about the effects of the Aswan Dam?
  • a)
    Crops cannot grow without silt.
  • b)
    Although farmers are pleased with the absence of seasonal flooding, they would prefer that the dam not have been built because of the problems it has caused.
  • c)
    The land around the river is not as fertile as it was before the dam was built.
  • d)
    Egypt now competes successfully on an economic level with Western nations.
  • e)
    The country would have been better off if the dam had not been built.
Correct answer is option 'C'. Can you explain this answer?

Inference questions require us to draw a conclusion based only upon the information presented in the passage.  
(A) The passage states that "before the dam” was built, blocking the flow of silt, “the Nile floodplain was famously productive." However, this choice goes too far by stating that crops “cannot” grow without silt. In fact, the passage indicates that farmers still grow crops on the land, despite the lack of silt in the soil.
(B) While it is reasonable to suppose that some farmers feel this way, the passage does not provide any information about the farmers' approval or disapproval of the dam.
(C) CORRECT.  In the second paragraph, the passage states "before the dam, the Nile floodplain was famously productive" and goes on to say that farmers now have to use artificial fertilizers.  This implies that the land is not as fertile as it was before the dam was built.  
(D) This idea was presented in paragraph one as a reason for the government to build the dam, but the rest of the passage does not address whether the government succeeded in this goal.
(E) This choice contradicts the last sentence of the passage, in which the author disagrees with those who think the dam should not have been built.

The ―paradox of tolerance‖ admonishes us that tolerance of the intolerant leads to intolerance. The Universal Declaration of Human Rights and the constitutions and laws of Western European democracies that adhere to the principle of freedom of speech all heed the warning of this conundrum and do not afford legal protection to extremist speech.  
While in Western European democracies, the speech of nondemocratic extremists has been successfully outlawed, in the United States the first amendment right to freedom of speech has been interpreted to encompass radical oration. The traditional justifications of this American stance originate in the belief that speech is entitled to greater tolerance than other kinds of activity. They are based on the belief that speech itself is valuable, and thus ascribe positive value to a very broad range of speech.
 According to the classical model, freedom of speech serves an indispensable function in the process of democratic self-government. From this perspective, the free speech principle need only protect political speech, comprised of all the facts, theories, and opinions relating to any issue on which the citizens must vote. Proponents of this view insist that even extremist views cannot be concealed from voting citizens, if these views bear on any public issue before them.
Protection of free speech serves the collective self-interests of a selfgoverning society made up of all rational, equal, and fully participating citizens who take their civic duties seriously. The fortress model is built on a foundation of pessimism, individualism, relativism, and self-doubt. At its deepest level, the fortress model values freedom of speech as a necessary precondition to the discovery and preservation of truth, but even at this level the function of speech remains primarily negative.  
From this perspective, the government and a majority of the people pose a great danger of intolerance. In spite of the high probability that their beliefs will eventually prove to be false, it is argued, people nonetheless tend to feel certain about them and, consequently, feel justified in requiring others to conform. Thus, the fortress model‘s prescription for combating the tendency to censor nonconforming views is to overprotect speech by providing a broad ―buffer zone‖ that encompasses extremist speech because its protection substantially diminishes the probability that inherently valuable speech will be suppressed.  
Directions: Read the above paragraph and answer the following
Q. All of the following actions have been put forth by one or another group in this country as being of value in our society. Which actions would violate a principle of the classical model of free speech?  
  • a)
    Banning an individual from making derogatory comments about various ethnic groups  
  • b)
     Banning an individual from yelling ―fire‖ in a crowded movie theater
  • c)
    Banning an individual from claiming that the government should be voted out of existence  
  • d)
    Banning an individual from making false statements about a company‘s products
  • e)
    Banning and individual for abusing his parents 
Correct answer is option 'C'. Can you explain this answer?

Mapping the Passage
¶1 introduces the ―"paradox of intolerance."
¶2 gives examples of countries that have taken the paradox into consideration by banning extreme speech, and argues that the United States tolerates even extremist speech.
¶3 introduces a justification for the defense of extremist speech: the classical model.
¶s4 and 5 introduce another justification: the fortress model.
Review the classical model in ¶3, paying particular attention to its scope. The classical model argues that political speech should be protected, whether it‘s extremist or not. While three of the answer choices deal with non-political speech, only (C) deals with political speech that the classical model would propose to
protect.
 

The person who, with inner conviction, loathes stealing, killing, and assault, may find himself performing these acts with relative ease when commanded by authority. Behaviour that is unthinkable in an individual who is acting of his own volition may be executed without hesitation when carried out under orders. An act carried out under command is, psychologically, of a profoundly different character than spontaneous action.
The important task, from the standpoint of a psychological study of obedience, is to be able to take conceptions of authority and translate them into personal experience. It is one thing to talk in abstract terms about the respective rights of the individual and of authority; it is quite another to examine a moral choice in a real situation. We all know about the philosophic problems of freedom and authority. But in every case where the problem is not merely academic there is a real person who must obey or disobey authority. All musing prior to this moment is mere speculation, and all acts of disobedience are characterized by such a moment of decisive action.  
When we move to the laboratory, the problem narrows: if an experimenter tells a subject to act with increasing severity against another person, under what conditions will the subject comply, and under what conditions will he disobey? The laboratory problem is vivid, intense, and real. It is not something apart from life, but carries to an extreme and very logical conclusion certain trends inherent in the ordinary functioning of the social world. The question arises as to whether there is any connection between what we have studied in the laboratory and the forms of obedience we have so often deplored throughout history. The differences in the two situations are, of course, enormous, yet the difference in scale, numbers, and political context may be relatively unimportant as long as certain essential features are retained.  
To the degree that an absence of compulsion is present, obedience is coloured by a cooperative mood; to the degree that the threat of force or punishment against the person is intimated, obedience is compelled by fear. The major problem for the individual is to recapture control of his own regnant processes once he has committed them to the purposes of others. The difficulty this entails represents the poignant and in some degree tragic element in the situation, for nothing is bleaker than the sight of a person striving yet not fully able to control his own behaviour in a situation of consequence to him.  
The essence of obedience is the fact that a person comes to view himself as the instrument for carrying out another‘s wishes, and he therefore no longer regards himself as culpable for his actions. Once this critical shift of viewpoint has occurred, all of the essential features of obedience—the adjustment of thought, the freedom to engage in cruel behaviour, and the types of justification experienced by the person (essentially similar whether they occur in a psychological laboratory or on the battlefield)—follow. The question of generality, therefore, is not resolved by enumerating all of the manifest differences between the psychological laboratory and other situations, but by carefully constructing a situation that captures the essence of obedience—a situation in which a person gives himself over to authority and no longer views himself as the cause of his own actions. 
Directions: Read the above paragraph and answer the following:
Q.In the context of the points being made by the author in the passage, the phrase ―absence of compulsion‖ (line 30) refers to: 
  • a)
    the lack of punishment in psychological experiments.  
  • b)
    obedience that is willingly given to one‘s superior.
  • c)
    the freedom to disobey the orders of those in authority.  
  • d)
    one‘s ability to consider the moral implications of an act.
  • e)
    having the free will to do what one wants 
Correct answer is option 'B'. Can you explain this answer?

In the passage, the author discusses the different factors that influence obedience, including the presence or absence of compulsion. The phrase "absence of compulsion" refers to a situation where obedience is not compelled by fear or the threat of force or punishment. Instead, it is colored by a cooperative mood, and individuals willingly give obedience to their superiors. This is supported by the statement in the passage that "to the degree that an absence of compulsion is present, obedience is colored by a cooperative mood." Therefore, option 2) is the correct answer.

Of course, in his attempts at field investigation, the historian is at the disadvantage that the countryside has changed in many respects since the period which he is studying. He is not permitted to use H.G. Wells‘s time machine, to enable him to see it as it actually was. Inevitably he is concerned in the main, if not exclusively, with literary and other materials, which have survived from that stretch of the past which interests him.  
Old maps may be plans of cities, charts of sea coasts and estuaries, cartularies of landed estates, or topographic delineations of land areas. These clearly engage the interest of historians and geographers alike, and they call for a combination of the methods and viewpoints of each. Maps can be conceived of and considered in several quite different ways, being properly regarded, and so assessed, as works of art—at best as objects of colour, skill, form, and beauty. They may alternatively be regarded purely for their cartographic aesthetic.  
The main queries which then arise are the following: how is it that the map-maker has carried out his task and with skill of what echelon and with what degree of success has he done so? Such an inquiry falls to the specialist field of historical cartography. An antiquarian map may also be approached in a means akin to that of the student who conceives it as a font contemporaneous with the time of its production. Thus, the historical cartographer may seek to bring grist to his mill and to consider the map‘s reliability as a satisfactory source of empirical evidence. By such means also the regional historian, in his search for essentials about such past matters as the availability of roads, the extent of enclosed farmland, or the number and location of mines and quarries, is no less an interested party.
 The value of old maps as documents useful for historicity depends necessarily on to what degree they depict and on how accurately. For virtually all periods of pre-modern history some maps have survived to serve as historiography, depicting, however imperfectly, certain features of past geography. The work of Claudius Ptolemy—who lived in the 2nd century A.D.—for centuries provided the basis for maps of the known world and its major regions. Although many were drawn on the scientific basis which he provided, they nevertheless embodied many errors—of location, distance, and the shape of areas of land and sea.  
The medieval portolan charts of the Mediterranean Sea and the later charts which provided sailing directions, produced in Holland, were accurate enough to be useful in practical navigation. Plans of important cities of Europe, so well-drawn as to yield evidence of their earlier form and extent, are notably offered in Braun and Hogenberg‘s Civitates Orbis Terrarum, published at Cologne and, in England, in John Speed‘s plans of cities. Similarly, John Ogilby‘s Britannia, Volume the First, appearing in 1675, gives detailed information of England's road system as it existed nearly three centuries ago. However, few of the early maps approach modern standards, which require accurate representation of distances and of heights above mean sea-level and the use of carefully distinguished symbols. This is because it was not until the 18th century that cartography, as an exact science, was born.  
Directions: Read the above paragraph and answer the following:
Q. According to the passage, which of the following statements is/are NOT true?  
I. Most maps produced before the 18th century are not as accurate as maps produced after the 18th century.  
II. The maps of Claudius Ptolemy were not used as a model by later mapmakers.
III. Historians have generally been uninterested in using maps as a tool to learn about the past. 
  • a)
    II only  
  • b)
    III only  
  • c)
    I and II  
  • d)
    II and III                  
  • e)
    I, II and III 
Correct answer is option 'D'. Can you explain this answer?

Arshiya Roy answered
Mapping the Passage
¶1 states that maps are valuable to historical research.
¶2 discusses the various traits of old maps and ways of studying them.
¶3 and 4 describe the value of old maps in relation to how much information they provide.
¶5 provides some examples of maps useful to the study of historical geography and describes the transition from pre-modern to modern maps.
Don‘t start with RN I to answer this question! It appears in only one choice and so isn‘t a time-effective starting point. RN II appears in three choices, so start there. RN II directly contradicts the author‘s point in ¶4 that Ptolemy‘s maps served as templates for other maps for centuries. Eliminate (B). RN III contradicts the main point of the passage: historians are interested in maps as historical tools. Since RNs II and III are both untrue, only choice (D) is possible. Though there‘s no need to check RN I, it can be verified as true by looking at the main point of the last paragraph.

The person who, with inner conviction, loathes stealing, killing, and assault, may find himself performing these acts with relative ease when commanded by authority. Behaviour that is unthinkable in an individual who is acting of his own volition may be executed without hesitation when carried out under orders. An act carried out under command is, psychologically, of a profoundly different character than spontaneous action.
The important task, from the standpoint of a psychological study of obedience, is to be able to take conceptions of authority and translate them into personal experience. It is one thing to talk in abstract terms about the respective rights of the individual and of authority; it is quite another to examine a moral choice in a real situation. We all know about the philosophic problems of freedom and authority. But in every case where the problem is not merely academic there is a real person who must obey or disobey authority. All musing prior to this moment is mere speculation, and all acts of disobedience are characterized by such a moment of decisive action.  
When we move to the laboratory, the problem narrows: if an experimenter tells a subject to act with increasing severity against another person, under what conditions will the subject comply, and under what conditions will he disobey? The laboratory problem is vivid, intense, and real. It is not something apart from life, but carries to an extreme and very logical conclusion certain trends inherent in the ordinary functioning of the social world. The question arises as to whether there is any connection between what we have studied in the laboratory and the forms of obedience we have so often deplored throughout history. The differences in the two situations are, of course, enormous, yet the difference in scale, numbers, and political context may be relatively unimportant as long as certain essential features are retained.  
To the degree that an absence of compulsion is present, obedience is coloured by a cooperative mood; to the degree that the threat of force or punishment against the person is intimated, obedience is compelled by fear. The major problem for the individual is to recapture control of his own regnant processes once he has committed them to the purposes of others. The difficulty this entails represents the poignant and in some degree tragic element in the situation, for nothing is bleaker than the sight of a person striving yet not fully able to control his own behaviour in a situation of consequence to him.  
The essence of obedience is the fact that a person comes to view himself as the instrument for carrying out another‘s wishes, and he therefore no longer regards himself as culpable for his actions. Once this critical shift of viewpoint has occurred, all of the essential features of obedience—the adjustment of thought, the freedom to engage in cruel behaviour, and the types of justification experienced by the person (essentially similar whether they occur in a psychological laboratory or on the battlefield)—follow. The question of generality, therefore, is not resolved by enumerating all of the manifest differences between the psychological laboratory and other situations, but by carefully constructing a situation that captures the essence of obedience—a situation in which a person gives himself over to authority and no longer views himself as the cause of his own actions. 
Directions: Read the above paragraph and answer the following:
Q. According to the passage, which of the following statements is NOT false?  
  • a)
    People will never commit acts that they judge to be wrong.  
  • b)
    People will always obey those who are in positions of authority over them.  
  • c)
    Obedience is not an important subject because it affects only a very limited number of acts.  
  • d)
    It is possible to study obedience through a laboratory experiment.                  
  • e)
    Obedience is not impacted by a cooperative mood 
Correct answer is option 'D'. Can you explain this answer?

Alok Iyer answered
¶1 states that people do things they otherwise wouldn‘t when so ordered by authority.
¶2 argues that psychological studies have to take into account the practical aspects of obedience in addition to theoretical ideas.
¶3 suggests that laboratory-tested obedience effectively highlights these practical aspects.
¶4 says that obedience is influenced by fear and the desire to cooperate, and that the individual obeying has trouble controlling his own behaviour.
¶5 expands on the point in ¶4: the laboratory can effectively simulate real-world conditions that lead to obedience.
Review the main points in the map, and read the stem carefully: you‘re looking for something that‘s not false, i.e., that is true. While three choices don‘t follow from the passage, (D) summarizes the point made in ¶3 that the lab is a good place to study obedience.

Measuring more than five feet tall and ten feet long, the Javan rhinoceros is often called the rarest large mammal on earth. None exist in zoos. Like the Indian rhino, the Javan has only one horn; African and Sumatran rhinos have two. While the Javan rhino habitat once extended across southern Asia, now there are fewer than one hundred of the animals in Indonesia and under a dozen in Vietnam. Very little is known about Javan rhinos because they lead secretive and solitary lives in remote jungles.
Until recently, scientists debated whether females even have horns, and most scientific work has had to rely on DNA garnered from dung.
The near extinction of the Javan rhino is the direct result of human actions. For centuries, farmers, who favored the same habitat, viewed them as crop eating pests and shot them on sight. During the colonial period, hunters slaughtered thousands. Now, human efforts to save them may well prove futile. The Vietnamese herd is probably doomed, as too few remain to maintain the necessary genetic variation. Rhinos from Java cannot supplement the Vietnamese numbers because in the millions of years since Indonesia separated from the mainland, the two groups have evolved into separate sub-species. In Indonesia, the rhinos are protected on the Ujung Kulon peninsula, which is unsettled by humans, and still have sufficient genetic diversity to have a chance at survival.
Ironically, however, the lack of human disturbance allows mature forests to replace the shrubby vegetation the animals prefer. Thus, human benevolence may prove little better for these rhinos than past human maltreatment.
Q.
The author’s attitude toward current human efforts to save the Javan rhino can best be described as
  • a)
    optimistic and worthwhile
  • b)
    pointless and doomed
  • c)
    idealistic but profitable
  • d)
    problematic and ironic
  • e)
    confused but heroic
Correct answer is option 'D'. Can you explain this answer?

Moumita Sen answered
In the second paragraph, after citing human actions as the cause of the Javan rhino’s plight, the author goes on to mention that the population in Vietnam is probably not viable and that human efforts in Indonesia, such as the protection of the rhino on the Ujung Kulun peninsula, have had mixed results. The passage ends with the thought that human benevolence is not helping the rhinos much more than past maltreatment. Thus, the correct answer will note the problems with current human efforts.  
(A) The author is certainly not optimistic, as the passage mentions that in Vietnam the outlook is doubtful and in Indonesia, problems plague the effort. It is possible, although not certain, that the author considers the effort worthwhile.
(B) The author gives no indication that he or she believes the effort is pointless. Furthermore, especially in Indonesia, the passage does not indicate a certainty that it will fail.
(C) The passage does not at all discuss the profitability of the effort. It is possible, although not certain, that the author considers the effort idealistic.
(D) CORRECT. The discussion of the mixed results and poor prognosis for the population in Vietnam justifies “problematic.” The last sentence of the passage, observing that benevolence has proved little better than maltreatment justifies “ironic.” Also, the author references the ironic quality of human protection of the Javan rhino in the 8th sentence in the 2nd paragraph, “Ironically, however, the lack of human disturbance allows mature forests to replace the shrubby vegetation the animals prefer.”
(E) The passage indicates that the efforts have been much less than entirely successful but does not indicate any confusion related to the current efforts. It is possible for efforts to be less than fully successful without being confused. Furthermore, it does not at all address the idea that any particular heroism is involved. 

The golden toad of Costa Rica, whose beauty and rarity inspired an unusual degree of human interest from a public generally unconcerned about amphibians, may have been driven to extinction by human activity nevertheless. In the United States, a public relations campaign featuring the toad raised money to purchase and protect the toad’s habitat in Costa Rica, establishing the Monteverde Cloud Forest Preserve in 1972. Although this action seemed to secure the toad’s future, it is now apparent that setting aside habitat was not enough to save this beautiful creature. The toad’s demise in the late 1980s was a harbinger of further species extinction in Costa Rica. Since that time, another twenty of the fifty species of frogs and toads known to once inhabit a 30 square kilometer area near Monteverde have disappeared.
The unexplained, relatively sudden disappearance of amphibians in Costa Rica is not a unique story. Populations of frogs, toads, and salamanders have declined or disappeared the world over. Scientists hypothesize that the more subtle effects of human activities on the world’s ecosystems, such as the build-up of pollutants, the decrease in atmospheric ozone, and changing weather patterns due to global warming, are beginning to take their toll. Perhaps amphibians - whose permeable skin makes them sensitive to environmental changes - are the “canary in the coal mine,” giving us early notification of the deterioration of our environment. If amphibians are the biological harbingers of environmental problems, humans would be wise to heed their warning.
Q. 
The author uses the adjective “subtle” in the second paragraph most probably to emphasize that
  • a)
    these effects are not easily recognized b y sophisticated testing equipment
  • b)
    these effects are difficult to notice because they take place over time on a global scale
  • c)
    these effects are so minimal that they affect only small animal species such as amphibians
  • d)
    these slight effects of human activity are rarely discussed by scientists
  • e)
    these effects are infrequently observed because they affect only specific world regions
Correct answer is option 'B'. Can you explain this answer?

Rajdeep Nair answered
In the second paragraph, the author mentions “the more subtle effects of human activity on the world's ecosystems,” and then lists three of these effects: the build-up of pollutants, the decrease in atmospheric ozone, and changing weather patterns due to global warming, all of which, scientists hypothesize, “are beginning to take their toll.” The key to answering this question correctly is to recognize, in context, what these effects have in common.
(A) Nowhere does the passage suggest or imply that these changes are not easily recognized by sophisticated testing equipment.  
(B) CORRECT. All of these effects happen on a global scale, so their immediate consequences on specific ecosystems are difficult to recognize. Furthermore, the passage states these effects are “beginning to take their toll,” indicating that the effects happen over time.
(C) Scientists hypothesize that these effects do affect amphibians, but nothing in the passage indicates that only small animal species such as amphibians are affected. In fact, the final sentence of the passage indicates that humans too might be affected by these environmental changes.
(D) The passage states that scientists hypothesize about the consequences of these subtle effects; this indicates that scientists do in fact discuss these effects
(E) The passage indicates that the consequences of these effects are global, causing disruptions in amphibious populations “the world over.” 

Measuring more than five feet tall and ten feet long, the Javan rhinoceros is often called the rarest large mammal on earth. None exist in zoos. Like the Indian rhino, the Javan has only one horn; African and Sumatran rhinos have two. While the Javan rhino habitat once extended across southern Asia, now there are fewer than one hundred of the animals in Indonesia and under a dozen in Vietnam. Very little is known about Javan rhinos because they lead secretive and solitary lives in remote jungles.
Until recently, scientists debated whether females even have horns, and most scientific work has had to rely on DNA garnered from dung.
The near extinction of the Javan rhino is the direct result of human actions. For centuries, farmers, who favored the same habitat, viewed them as crop eating pests and shot them on sight. During the colonial period, hunters slaughtered thousands. Now, human efforts to save them may well prove futile. The Vietnamese herd is probably doomed, as too few remain to maintain the necessary genetic variation. Rhinos from Java cannot supplement the Vietnamese numbers because in the millions of years since Indonesia separated from the mainland, the two groups have evolved into separate sub-species. In Indonesia, the rhinos are protected on the Ujung Kulon peninsula, which is unsettled by humans, and still have sufficient genetic diversity to have a chance at survival.
Ironically, however, the lack of human disturbance allows mature forests to replace the shrubby vegetation the animals prefer. Thus, human benevolence may prove little better for these rhinos than past human maltreatment.
Q.
The author states that which of the following was most responsible for the near extinction of the Javan rhino?
  • a)
    farmers shooting them on sight
  • b)
    the separation of Indonesia from the mainland
  • c)
    hunters slaughtering thousands
  • d)
    current human efforts to save them
  • e)
    the cumulative effect of many past human activities
Correct answer is option 'E'. Can you explain this answer?

In the second paragraph, the passage states that the “near extinction of the Javan rhino is the direct result of human actions.” The correct answer must be justified by that statement.  
(A) Rhino deaths at the hands of farmers are mentioned as a detail and as just one of the human actions responsible. It does not single farmers out as the primary cause of the Javan rhino’s near extinction.
(B) The passage mentions the separation of from the mainland to explain why the Javan rhino population in Vietnam cannot be supplemented by rhinos from Ujung Kulon. It is not discussed as a reason for the near extinction of the Javan rhino.
(C) Rhino deaths at the hands of hunters are mentioned as a detail and as just one of the human actions responsible. It does not single hunters out as the primary cause of the Javan rhino’s near extinction.
D) The passage does not say that current human efforts are at all responsible for the near extinction of the Javan rhino.  Instead, it indicates that current benevolent human efforts may not prove sufficient to save them.
(E) CORRECT. The beginning of the second paragraph clearly states that human actions, taken as a whole, have directly caused the near extinction of the Javan rhino.

Direction For question : As the Heart of Darkness makes its way into the savage shadows of the African continent, Joseph Conrad exposes a psycho-geography of the collective unconscious in the entangling symbolic realities of the serpentine Congo. Conrad’s novella descends into the unknowable darkness at the heart of Africa, taking its narrator, Marlow, on an underworld journey of individuation, a modern odyssey toward the center of the Self and the center of the Earth. Ego dissolves into soul as, in the interior; Marlow encounters his double in the powerful image of ivory-obsessed Kurtz, the dark shadow of European imperialism. The dark meditation is graced by personifications of anima in Kurtz’ black goddess, the savagely magnificent consort of the underworld, and in his porcelain -skinned Persephone, innocent intended of the upper world. “Africa,” wrote Graham Greene, “will always be the Africa in the Victorian atlas, the blank unexplored continent in the shape of the human heart.” The African heart described by Greene “acquired a new layer of meaning when Conrad portrayed the Congo under King Leopold as the Heart of Darkness, a place where barbarism triumphs over humanity, nature over technology, biology over culture, id over super ego.” The unknown and uncharted topography of the African continent first beckoned Conrad’s narrator, Marlow, into its depths in his boyhood: “Now, when I was a little chap I had a passion for maps. I would look for hours at South America, or Africa, or Australia, and lose myself in all the glories of exploration”. When Marlow was grown and Africa was no longer a blank space on the map, but rather “a place of darkness,” there was still one river there that drew him especially, “a mighty big river, that you could see on the map, resembling an immense snake uncoiled, with its head in the sea, its body at rest curving afar over a vast country, and its tail lost in the depths of the land”. This same deep place, the Congo, that had seduced Conrad’s ivory hunting Kurtz into the horrors of its savage embrace had, in 1890, lured Conrad himself into adventure that turned him from sailor to writer and severely affected his health for the rest of his life. As the voyage up the Congo pro ved fateful for the development of Conrad’s narrator, Marlow, it was equally fateful for Conrad’s individuation, as he reflects in his letters “Before the Congo I was just a mere animal.” Hillman, in “Notes on White Supremacy” reminds us that, like Conrad, both Freud and Jung were called to venture into the shadowed continent and vestiges of their journeys still colour our psychological language: The convention informing geographical discoveries and the expansion of white consciousness over Africa continue to inform psychic geography. The topological language used by Freud for “the unconscious” as a place below, different, timeless, primordial, libidinal and separated from the consciousness recapitulates what white reporters centuries earlier said about West Africa. From Conrad’s Heart of Darkness to van der Post’s Venture to the Interior, Africa and the unconscious allegorize the other place.... “Just don’t stay in the topical colonies too long; you must reign at home,” writes Freud in 1911 to Jung, who himself made the African journey fourteen years later, describing the vast lands and dark peoples he encountered in language he applies as well to the immemorial unconscious psyche.... Part of psychology’s myth is that the unconscious was “discovered” as its contents are “explored”. Thus Africa has become a topology of the mind — its location, its shape, its cultures, its textures, its rhythms, its foliage, its hues, its wildness — all calling forth something lost in the psychology of the white European. It is with an understanding of our destiny to explore that symbolic lost continent within ourselves that we can begin to appreciate the prescience of Jungian psychology in Conrad’s Heart of Darkness.
Q. Which of the following is not in alliance with the representation of ‘Africa’ as in Conrad’s Heart of Darkness?
(a) Conrad portrays Africa (Congo) as being inhibited by people who are primitive and uncivilized.
(b) Civilization is shown to be non-existent in the depths of Africa.
(c) Africa is portrayed as a place full of delights and charms, which attracts the attention of Marlow.
  • a)
    Both a and c
  • b)
    a, b, and c
  • c)
    Only c
  • d)
    Only b
Correct answer is option 'C'. Can you explain this answer?

Aditya Kumar answered
Option (3) is the correct answer. Statements (a) and (b) can be inferred from the passage. In paragraph 2, the author calls Africa as “a place where barbarism triumphs”. Barbarism points towards presence of uncivilized people and lack of civilization. Statement (c) is the most appropriate as it is nowhere stated in the passage. What Marlow was attracted to, was a river in Africa. Calling Africa as a place of delights and charms will be too mainstream.

Direction for question: Typewriters are the epitome of a technology that has been comprehensively rendered obsolete by the digital age. The ink comes off the ribbon, they weigh a ton, and second thoughts are a disaster. But they are also personal, portable and, above all, private. Type a document and lock it away and more or less the only way anyone else can get it is if you give it to them. That is why the Russians have decided to go back to typewriters in some government offices, and why in the US, some departments have never abandoned them. Yet it is not just their resistance to algorithms and secret surveillance that keeps typewriter production lines – well one, at least – in business (the last British one closed a year ago). Nor is it only the nostalgic appeal of the metal body and the stout well-defined keys that make them popular on eBay. A typewriter demands something particular: attentiveness. By the time the paper is loaded, the ribbon tightened, the carriage returned, the spacing and the margins set, there’s a big premium on hitting the right key. That means sorting out ideas, pulling together a kind of order and organising details before actually striking off. There can be no thinking on screen with a typewriter. Nor are there any easy distractions. No online shopping. No urgent emails. No Twitter. No need even for electricity – perfect for writing in a remote hideaway. The thinking process is accompanied by the encouraging clack of keys, and the ratchet of the carriage return. Ping!
Q. According to the passage, some governments still use typewriters because:
  • a)
    they do not want to abandon old technologies that may be useful in the future.
  • b)
    they want to ensure that typewriter production lines remain in business.
  • c)
    they like the nostalgic appeal of typewriter.
  • d)
    they can control who reads the document.
Correct answer is option 'D'. Can you explain this answer?

Aditya Kumar answered
“Type a document and lock it away and more or less the only way anyone else can get.......... some departments have never abandoned them”. This information, which says that using of typewriters helps one control who views the document, makes option 4 correct.

Chapter doubts & questions for Reading Comprehension for UPPSC - Course for UPPSC Preparation 2025 is part of UPPSC (UP) exam preparation. The chapters have been prepared according to the UPPSC (UP) exam syllabus. The Chapter doubts & questions, notes, tests & MCQs are made for UPPSC (UP) 2025 Exam. Find important definitions, questions, notes, meanings, examples, exercises, MCQs and online tests here.

Chapter doubts & questions of Reading Comprehension for UPPSC - Course for UPPSC Preparation in English & Hindi are available as part of UPPSC (UP) exam. Download more important topics, notes, lectures and mock test series for UPPSC (UP) Exam by signing up for free.

Course for UPPSC Preparation

113 videos|360 docs|105 tests

Top Courses UPPSC (UP)

Signup to see your scores go up within 7 days!

Study with 1000+ FREE Docs, Videos & Tests
10M+ students study on EduRev